You are on page 1of 63

Physical Sciences Grade 12

2018 Memoranda
Table of Contents

Page

1. General Introduction I – no exercises

2. General Introduction II – no exercises

3. Mechanics I 1
 Revising Grade 10 / 11 Work
 Newton’s Laws of Motion

4. Mechanics II 5
 Momentum & Impulse
 Vertical Projectile Motion & Graphs

5. Mechanics III 11
 Work, Energy and Power

6. Waves, Sound and Light I 15


 Properties of Sound
 The Doppler Effect

7. Electricity and Magnetism I 19


 Electrostatics – Coulomb’s Law, Electric Fields, Ohm’s Law,
Power & Energy
 Electric Circuits – Internal Resistance, Series and Parallel Networks.

8. Electricity and Magnetism II 23


 Electrical Machines
 AC current

9. Matter and Materials I 27


 Optical Phenomena – the photoelectric effect, emission
and absorption spectra

10. Matter and Materials II 31


 Organic Molecules & Macromolecules

11. Matter and Materials III 35


 Organic Reactions, Intermolecular Forces, Physical Properties

12. Chemical Change I 39


 Representing Chemical Change
 Quantitative Aspects

13. Chemical Change II 43


 Energy and Change – Energy and change, Rates of Reaction,
Chemical Equilibrium, Factors
14. Chemical Change III 49
 Acids and Bases

15. Chemical Change IV 53


 Electrochemistry – electrolytic & galvanic cells, electrode
potentials, redox reactions

16. Chemical Systems 57


 Chemical Industry – Nutrients and Fertilisers, Industrial
Processes and the Environment.
Grade 12
Exam Preparation

Mechanics I
Memorandum

TouchTutor® Series
Physical Sciences

Section A (Multiple Choice Questions)


1) B – F = ma, therefore F α a, with m constant.
2
2) D – the gravitational force α 1/d , thus halving the distance implies a force that is quadrupled.
3) B – the kinetic energy decreases. The diagram shows the frictional force as larger than the applied force,
i.e. with a resultant force to the left (or west), slowing the object down. Remember that frictional force is
always in a direction opposite to the motion of the object.
4) D – equally great. Pushing the crate, you have to overcome the force of friction, f k = μN (with N = mg),
which is independent of the surface contact area.
5) C – ∆ 𝑣⃗ = 𝑣⃗𝑓 −𝑣⃗𝑖 = 1,94 − (+1,93) = 0,01 𝑚 · 𝑠 −1 to the left.
2 3
6) C – W net = ∆Ek = F∆xcos(θ), therefore ½ (500)(22,22) = F × 50 × cos(180°); ∴ F = -2,468 × 10 N, in
-1 -1
direction opposite to car’s motion. Remember to convert km·h to m·s Note: answered here using the
work-energy theorem, but can also be answered using Newton’s laws of motion.
7) D – from 0 to 8 seconds, the graph shows a constant increase in velocity, i.e. an acceleration.
𝑎 = ∆𝑣⁄∆𝑡 = = 5 𝑚 · 𝑠 −2
40
8
2
8) A – speed. Ek = ½ mv – note, the object does not have to be accelerating.
3
9) This question is best answered using some calculus learnt in your maths class: a = dv/dt = d(4 + 2t )/dt =
2 -2
6·t . At 2 s, a = 24 m·s . An alternative way is to calculate the velocity a brief time before 2 s, and also

after 2 s. Let’s take t = 1,9 s, and 2,1 s.


At t = 1,9 s: v = 4 + 2(1,9)3 = 17,718
At t = 2,1 s: v = 4 + 2(2,1)3 = 22,522
∆𝑣 22,522 − 17,718 4,804
𝑎= = = = 24,02 𝑚 · 𝑠 −2
∆𝑡 2,1 − 1,9 0,2
Correct answer: D..
10) E – T2 will be equal to the horizontal component of T3; the vertical component of T3 is equal to T1 (W), the
weight (mg) of the object.
T2: tan(60°) = opp./adj. = W/T2, therefore T2 = W / tan 60°
T3: sin (60°) = opp./hyp. = W/T3, therefore T3 = W / sin 60°
11) B – II and III only. (I) The rate at which objects fall (their gravitational acceleration) depends on the mass
of the planet (g = GM/r2). Since the masses differ, the gravitatioanl acceleration will differ too. (II) For an
equal-arm balance, the gravitational attraction on both masses being weighed is the same, so this is
independent of the size of the gravitational attraction on different planets. (III) impulse = change in
momentum; F∆t = m∆v, and since both m and v are independent of the gravitational attraction of the
particular planet, this is the same for both.
-2 2 2 2 2 -2
12) B – 4 m·s . gE = GME/r ; gM = GMM/d = G(0,1)ME/(0,5r) = 0,4 × GME/r = 0,4 × 9,8 = 3,92 m·s
2 2 2
13) B – 3/4 W. W E = GmME/R ; WP = GmMP/RP = 3GmME/4R = ¾ W E.
pg 1
14) C – distance may be calculate from a velocity-time graph by
calculating the area under the graph. From t = 0 to 1 s, the object
travels distance (the orange area) in a negative direction. From t =
1 s onwards, the objects travels a distance (the green area) in a
positive direction. When the sum of the orange and green areas =
zero, then the object passes its initial position again.
st
15) C – – 36 m. For the 1 second: ∆x = vt = 24(1) = 24 m. From 1
th 2 2
– 11 s: ∆x = vi∆t + ½ a(∆t) = 24(10) + ½ (-6)(10) = – 60 m. The position of the body at t = 11 s is 24 –
60 = –36 m (i.e. 36 m to the left of the starting position – assuming to the right is positive)
16) C – T1 < T2. As T2 is more vertical, it is supporting more of the weight of the ball. The horizontal
components of T1 and T2 are equal.
17) E – diagrams A and C, with W not vertical, are clearly wrong. Diagram D has friction in the wrong direction
and doesn’t know what the normal force is. Diagram B also has the normal force wrong.
rd
18) C – they act on different bodies. A very important aspect of Newton’s 3 law.
19) C – 375 N. Total downward force = 125 + 500 = 625 N. Total upward must be the same,
therefore 625 N, made up of 250 N on the right and 375 N on the left.
20) C – 6,4 kN up. FR = ma = 800 × 2 = 1 600 N. FR = Fup + W, therefore Fup = 1600 N –
-2
800·9,8 = – 6240 N (or if g = 10 m·s , Fup = -6400 N)

Section B (Structured Questions)


Question 1
1) If a resultant force acts on a body, it will cause the body to accelerate in the direction of the resultant
force. The acceleration of the body will be directly proportional to the resultant
force and inversely proportional to the mass of the body.
2) The coefficient of kinetic friction, μk, remains constant since it is only dependent
on the two materials that are in contact and the nature of their surfaces.
3) There are 4 forces acting on the 6 kg mass: the force due to gravity (𝐹⃗𝑔 ) –
always acting vertically downwards, the normal force (𝐹⃗𝑁 𝑜𝑟 𝑁 ⃗⃗ ) – perpendicular
to the surface, the frictional force (𝐹⃗𝑓 ) – opposing the motion, and the tension
(𝐹⃗𝑇 𝑜𝑟 𝑇
⃗⃗) in the string.
4) (i) take up the slope as positive; we are interested in the forces (or the components of these forces) in
-2
the line of motion – their sum = resultant force that produces an acceleration of 4 m∙s .
F  m  g  6  9, 8  58, 8 N

 
g

F ( N )  F  cos(30)  58, 8  3  50, 922 N


N g
2
F   N  0, 2  50, 922  10,184 N
f k

Now 𝐹⃗𝑇 + 𝐹⃗𝑓 + 𝐹⃗𝑔|| = 𝐹⃗𝑛𝑒𝑡 = 𝑚𝑎⃗


therefore 𝐹⃗𝑇 + (-10,184) + (–58,8 × sin(30°)) = 6 × 4, therefore 𝐹⃗𝑇 = 63,584 N up the slope
(ii) take up the slope as positive … given a constant velocity, the acceleration of the system (𝑎⃗) = 0.
Note: because it is a light inelastic (or inextensible) string, the forces between
the two masses are internal forces and so may be ignored (they cancel each
other); alternately, consider the two masses connected side by side.
What forces are acting on the two-body system: (i) the force F, (ii) the frictional
forces (both bodies), (iii) the component of the gravitational force in the direction
of motion (again for both forces) – thus
Fnet  ma  F  Ff 6  Ff 3  Fg ||6  Fg ||3
 (6  3)(0)  F  (  0, 2  6  9, 8  cos(30))  (  0,1  3  9, 8  cos(30))
 (  6  9, 8  sin(30))  (  3  9, 8  sin(30))
 F  0  10,184  2, 546  29, 4  14, 7  56, 83 N
5) The tension in the string would decrease since there is no friction that the upward force needs to
overcome.

Question 2

pg 2
-3 -1
1) velocity at B = velocity of interval AC = distance A-C / time = (14 × 10 )/(2 × 0,02) = 0,35 m·s
-3 -1
2) velocity at G = distance F-H / time = (34 × 10 )/(2 × 0,02) = 0,85 m·s
-2
3) average acceleration from B to G = change in velocity / time interval = (0,85 – 0,35)/(5 × 0,02) = 5 m·s

Question 3
Fnet = sum of forces on P
= FS on P + FR on P
= 0,6 + (– 0,2) = 0,4 N.
Fnet = 0,4 N towards S.

Question 4
2 -11 2 -10
1) F = Gm1m2/r = 6,67×10 × 8 × 6 / 2 = 8,004 ×10 N - always an attractive force
2 -11 6 2 24
2) F = GmEm1/r ∴ 9,8 = 6,67×10 × mE × 1 / (6,4×10 ) ∴ mE = 6,018 ×10 kg
3)
2 -11 -31 -10 -10 2 -30
a) Fg = Gmemp/r = 6,67×10 × 9,1×10 × 1,7×10 / (0,59×10 ) = 2,964×10 N
2 9 -19 -19 -10 2
b) FE = kQ1Q2 / r = 9,0×10 × 1,6×10 × 1,6×10 / (0,59×10 )
- clearly, electrostatic forces are much stronger than gravitational forces.

Question 5
2 -11 24 30 11 2 8 2 22
1) F = GmEmS/r = 6,67×10 ( 5,98×10 × 1,99×10 ) / [(2×10 ) + (4×10 ) ] = 1,984×10 N
use Pythagoras’ theorem to calculate the distance from the sun to the earth – the expression within
the square brackets above.
2 2 -11 22 6 2 -2
2) F = m1g = G(mMm1/r ) ∴ g = GmM / r = 6,67×10 × 7,35×10 / (1,6×10 ) = 1,915 m·s
-3
3) Weight = mg = 50 × 10 × 9,8 = 0,49 N

Question 6
1) Velocity / time = acceleration. The physical quantity represented by the gradient is acceleration.
-2
2) gradient = Δy / Δx = (94 – 24) / (2 – 0,5) = 46,67 m·s
3) i) constant acceleration ii) the car is slowing down (neg. acc.) but still a positive velocity.
4) distance = velocity × time (the area under the graph) = ½ (2)(94) = 94,0 m
-1 -1 -1
5) i) at 4 s, the speed of the car is 136 m·s ii) 136 m·s = 136 × 3600/1000 = 489,6 km·h -
which is an improbable speed, even for a F1 racing car.
6) Calculate the acceleration of the car, then the force …

v f  vi  2a  y   0  145  2a (150)  a  70, 083 m  s


2 2 2 -2

Fnet  ma  (1000)(70, 083)  70083,33 N


The force acts in the direction opposite to the motion of the car.
7) This is an application of the Doppler effect. As the car (D) moves to
the right, away from the listener (X), the pitch of the sound (its
perceived frequency) will decrease.
8) A shockwave is formed when an object breaks the sound barrier, i.e. when it travels faster than the
-1
speed of sound. If the F1 car travels at 375 km/h, it travels at 375 × 1000/3600 = 104,167 m·s . This is
-1
much slower than the 340 m·s at which sound travels, so the F1 car will not cause a shockwave.
Question 7
1) If a resultant force acts on a body, it will cause the body to accelerate in the direction of the resultant
force. The acceleration of the body will be directly proportional to the resultant force and
inversely proportional to the mass of the body. The mathematical representation is: 𝐹⃗ = 𝑚𝑎⃗
2) see diagram to righyt
3) Calculate the resultant forces, using the formula Fnet  ma - take upward as positive
Calculate T1 as it applies to the 5 kg and the 20 kg block … first for the 5 kg block …
T1  T2  Fg  ma  (T1 )  250  (5)(9, 8)  5a  T1  201  5a

pg 3
Note: T1 acts downwards, hence the substitution –T1 (magnitude and direction) above.
Now calculate T1 as it applies to the 20 kg block … remember, T1 here acts in a positive direction.
T1  Fg  ma   T1  (20)(9, 8)  20a  T1  196  20a
Having calculated the magnitude of T1 for both block, we can equate them (it’s the same string, after
all) – thus 196  20a  201  5a  25a  5  a  0, 2 m  s
-2

Substitute into any T1 equation: T1 = 196 + 20(0,2) = 200 N


The magnitude of the tension in P is 200 N.
4) If given a sharp pull, Q will break rather than P. It has to do with the inertia of the blocks … When the
string is pulled sharply, it will stretch to breaking point before the 5 kg mass (given its inertia) has
moved enough to stretch string P.

Question 8
1) See diagram to right
nd
2) Apply Newton’s 2 Law: Fnet = ma
For 2 kg block .. (taking upward as positive)
T – Fg1 = m1a, therefore T – 2×9,8 = 2a
For 4 kg block … (taking down as positive)

–T + m2g = m2a, therefore –T + 4×9,8 = 4a


Solve for a …
-2
T – 19,6 – T + 39,2 = 6a, therefore 6a = 19,6 and thus a = 3,27 m·s
3) Substitute for a in equation for 2 kg block, then
T – 19,6 = 2×3,27, thus T = 26,14 N
4) The net force on m1 is smaller than the net force on m2.
5) The pulley will turn anti-clockwise.

Question 9
1) Equilibrium exists in this system because the resultant force is zero
2) An error might occur in the plotting of the position of the strings due to the
parallax error ..
3) Each 100 g weight exerts a force of 100×10-3 × 9,8 = 0,98 N on knot O.
4) The force due to gravity on mass m is balanced by the vertical component of
each of the two upward forces, i.e. the sum of forces = 0
Vertical component of each upward force = 0,98 × sin(45°) = 0,693 N
Thus downward force = mg = 2 × 0,693 = 1,386 N, and thus m = 1,386/9,8 =
0,14143 kg or 141,43 g.

pg 4
Grade 12
Exam Preparation

Mechanics II
Memorandum

TouchTutor® Series
Physical Sciences

Section A (Multiple Choice Questions)


1) D – conservation of momentum. Momentum before = zero, therefore sum of momentum of P and q after
the spring release must also = zero.
2) B – impulse (F∆t) = m∆v = ∆p – the change in the momenum of a body.
-1
3) A – 5,7 m·s . Momentum before = momentum after, therefore …
(7,0)v + (2)(0) = (7,0)(4) + (2)(6,0), ⇒ v = (28 + 12)/7 = 5,7
4) Note: this question requires lots of mathematical manipulation and you are unlikely to get a question like
this in your matric exam. Work through the answer, and try to understand it.
First apply the conservation of energy: Total Ek before = total Ek after, therefore …
2 2 2
½ m(vA) + 0 = ½ m(vA/3) + ½ MB(vB) ― (if kinetic energy of A decreases by 1/9, then the velocity is
1/3 of the original)
2 2 2 2 2
∴ m(vA) = m(vA) /9 + MB(vB) ⇒ 8m(vA) /9 = M(vB) - equation ①
Now apply the conservation of momentum: Total p before = total p after, therefore …
mvA + o = – m(vA)/3 + MvB ― remember: vA after collision = 1/3 vA before, and in opp. direction
∴ 4mvA/3 = MvB, ⇒ vA = 3MvB/4m – equation ②
Now substitute equation ② into equation ①
2
3𝑀𝑣𝐵⁄
then 8𝑚⁄9 × ( 2
4𝑚) = 𝑀(𝑣𝐵 ) ⇒ M/2m = 1, therefore M = 2m. Correct answer: D

5) B – an elastic collision is defined as one in which the total kinetic energy is conserved
6) A – the ball accelerates uniformly, thus its velocity increases at a steady rate, and therefore its momentum
increases at a steady rate. With downward as positive, the momentum will increase constantly until
impact with the ground.
7) D – linear momentum is conserved. Note, just because the external forces are zero does not mean that
energy is constant. Think of a swing. When under its own motion, the potential energy is converted to
kinetic energy and back. Energy might also be lost, as in an inelastic collision.
8) B – the maximum height = 66,1 m. First calculate the velocity of the ball after the vertical impulse. F∆t =
-1
m∆v, so 18 = (0,5) × ∆v, therefore ∆v = 36 m·s . Now use equations of motion to determine max.
height: v f  vi  2a (y ) , ∴ ∆y = 36 /(2 × 9,8) = 66,1 m
2 2 2

-1 -1
9) B – 5,2 m·s m1v1 + m2v2 = (m1 + m2)v, therefore v = (25 × 6)/(29) = 5,17 m·s
10) B – apply the conservation of momentum …
Momentum before: (M + m)v / after: mvp — they are equal … therefore vp = (M + m)v / m
11) A – Momentum (𝑝⃗) = m𝑣⃗, i.e. both are vector quantities, and the direction of momentum is always the
same as the direction of the velocity. Remember that the particle can be moving faster or slower, kinetic
energy is a scalar quantity (no direction), and the net force can be in the direction opposite to the motion
when it is slowing down.
12) A – a projectile, even as it changes direction, is under the influence of gravitational attraction, with a = -9,8
-2
m·s if upward is the positive direction

pg 5
13) C – acceleration. Both spheres experience the same gravitational attraction, which is independent of the
mass of the object being attracted. All the other concepts, momentum, kinetic and potential energy are all
mass dependent.
14) B – a projectile is defined as moving under the influence of gravity only, and gravity acts towards the
Earth’s centre, even while the projectile is moving vertically upward.
15) C – vi = -vf ∴ starting and end position the same. Gravitational acceleration applies, therefore
position/time not a straight line.
16) D – in this graph, downward is taken as positive, and the initial position, before dropping, is the zero
position. A represents the point at which the ball reaches the ground the first time, then it bounces back up
to B and hits the ground a second time at C. D is the max. height after the second bounce.
17) B – 2 s
-1 -2 -1
18) vf = vi + at, with vf = 0 m·s , t = 2 s and a = -9,8 m·s . Therefore vi = 0 – (-9,8)(2) = 19,6 m·s
19) C – the slope of a distance vs time graph represents velocity (v = ∆x/∆t), and if the line is straight, it is a
constant velocity.
20) A – the ball is thrown downwards, hence its velocity increases (hence not C or D). The downward velocity
just before hitting the ground is the same as the upward velocity just after the bounce (assuming an elastic
collision), hence not B.

Section B (Structured Questions)


Question 1
1) The total momentum of an isolated system is conserved.
2) To right is positive: Total momentum before = total momentum after
mb vb  m p v p  m( b  p ) v f  (75)(3)  (2)( 5)  (77) v f

 v f  2, 792 m  s (to right)


-1

Question 2
1) a) acceleration is upwards due to rocket engines b) with rocket fuel used up, the rocket is a projectile
moving only under the influence of gravity, i.e. acceleration is downwards.
2) At point Q – the only force acting on the rocket (ignoring air resistance) is the force due to gravity.
3) Apply equations of motion from moment rocket fuel is used up .. with upward as positive, initial velocity =
-1
+112,8 m∙s , Δy (from start when fuel is used up to striking ground) = -225,6 m, acceleration = g = -9,8
-2
m∙s . To avoid solving quadratic equations, first calculate the final velocity.
v f  vi  2a  y   (112, 8)  2(9, 8)(225, 6)  17145, 6  v f  130, 941 m  s
2 2 2 -1

Now calculate the time taken, then add the 4 seconds for the first part of the journey ….
 v f  vi   130, 941  112, 8  t  t  24, 872 s
y    t   225, 6   
 2   2 

Total time taken = 24,872 + 4 = 28,872 s


(As alternative, you can calculate time taken to maximum height
(11,51 s), and the maximum height (874,78 m), and use these
values to calculate the time taken from max. height to striking the
ground (13,36 s). Adding these time intervals, with the original 4
s under rocket power, gives total time taken as 28,87 s).
4) Note: when fuel is used up, the rocket is a projectile, thus
acceleration is constant (graph must be a straight line with
-2
gradient = acceleration of –9,8 m·s .

Question 3
1) The momentum of a body is the product of its mass and velocity:
𝑝⃗ = 𝑚𝑣⃗
2) For the time interval 10 – 20 s, the momentum of the body does not change, i.e. ∆𝑝⃗ = 0 (the
gradient = 0)
∆𝑝⃗
but 𝐹⃗𝑛𝑒𝑡 = ⁄∆𝑡 = 0⁄10 = 0 , thus the statement is true.

3) Impulse = change in momentum … Fnet  t  p  p f  pi  120  (50)  170 N  s


-1
The magnitude of the change in momentum is 170 kg·m·s .

pg 6
4) Assuming an isolated system, the total momentum of the system is conserved, i.e. the total momentum
before = total momentum after the collision, take direction east as positive
piA  piB  p fA  p fB   120  70  50  p fB  p fB  100 kg  m  s
-1

-1
Object B has a momentum of 100 kg·m·s travelling west.

Question 4 1,84
1) Maximum height is reached when the velocity of the object comes to zero
.. i.e. after 1,84 seconds.
2) Height above the starting point = area under the graph up to 1,84 s. Thus
height = ½ (18)(1,84) = 16,56 m
3) First calculate the time is takes the object to fall from max. height to the
ground … take upward as positive …
v f  vi  at  42  0  (9, 8)t  t  4, 286 s
Total time from launch = 1,84 + 4,286 = 6,126 s.
Distance object falls downwards = y  vi t  12 a  t   0  12 (9, 8)(4, 286)  90, 01 m or 90,01 m
2 2
4)
downward. If object travelled 16,56 m upward first, the height of the launch must be equal to 90,01 –
16,56 = 73,45 m

Question 5
a) Change in momentum = m(v f  vi )  5500(7, 8  4, 2)  1, 980  10 kg  m  s
4 -1
1)

p 1, 98  10
4

b) Fnet    1, 32  10 N
3

t 15
-1 -1 -1 -1
2) convert cm·s into m·s – 63 cm·s = 0,63 m·s
Fnet t  m(v f  vi )  (35)t  (72  10 )(0, 63)  t  1296 s  21, 6 min
3

3) a) Assume an isolated system and the conservation of momentum … take towards tryline as positive
m1v1  m2 v2  m3v3   m1  m2  m2  v  (92)(5)  (75)(2)  (75)(4)  242v  v  0, 041 m  s
-1

b) Yes, a try should be scored – the diving player is still moving forward and should cross the line.

Question 6
1) Free fall occurs when an object moves under the influence of gravity alone (i.e. excluding air resistance) –
it is also called projectile motion. Note that the object can move upwards during free fall, as when a ball is
thrown upwards. As soon as the ball leaves the thrower’s hand, it is in free fall.
2) Take upward as positive – there are different, equally valid, ways of calculating the time …
Projectile motion is symmetrical – the upward velocity when thrown = downward velocity at same point or
height … thus v f  vi  at   15  15  (9, 8)t  t  3, 061 s

Ball A reaches maximum height at 1,53 s: y  vi t  12 a  t   (15)(1, 53)  12 (9, 8)(1, 53)  11, 48 m
2 2
3)

- at this point, ball B has fallen downwards: y  vi t  12 a  t   0  12 (9, 8)(1, 53)  11, 47 m - i.e.
2 2

11,47 m downwards
The distance between A and B then is 11,48 + 11,47 = 22,95 m
4) taking upward as positive …

Question 7
-2
1) take up as positive, then g = -9,8 m·s and Δy = -15 m
v f  vi  2a  y   0  2(9, 8)(15)  294  v f  17,15 m  s
2 2 -1

2) Yes, it would change. The velocity would be lower since the force
due to air resistance (friction) opposes the downward motion due to
gravity.
3) a) take downward as positive, as instructed
Before calculation the impulse, we need to know the velocity with which the cricket ball rebounds off
the floor. Remember, upward displacement now is negative.
v f  vi  2a  y   0  vi  2(9, 8)(4)  vi  8, 854 m  s
2 2 2 -1

pg 7
Now calculate the impulse … Fnet·Δt
3
Fnet t  p  m(v f  vi )  (156  10 )(8,85  (17,15))  4, 056 kg  m  s
-1

-1
The change in momentum is 4,056 kg·m·s upwards
b) Fnet  p t  4,056 0,8  5, 07 N - or 5,07 N upwards
4) taking downwards as positive, and taking point A as zero position …
Note: this graph looks different from the official memo where ground
was taken as the zero position.
5) A softer ball will deform on impact with the ground and thus stay in
contact with the ground for a longer period of time. If the change in
momentum of the softer ball is the same as for the harder cricket ball,
then the force must decrease given Fnet  p t

Question 8
1) projectile motion or free fall
2) a) take downward as positive .. then use equations of motion …
y  vi t  12 a(t )2  (30)  vi (1,5)  12 (9,8)(1,5)2  vi  12,65 m  s-1
b) The velocity at B is now the final velocity … initial velocity = 0 (again, take down as positive)

v 2f  vi2  2a(y)  (12,65)2  0  2(9,8)y y  8,16 m


Point X is 8,16 m above B and thus 8,16 + 30 = 38,16 m above ground.
3) Note – you’re asked to draw an acceleration-time graph – the acc. is a
-2
constant 9,8 m·s for the whole motion, except the moment of the
bounce. Indicate this via a dotted line. In this graph, downward is taken
as positive.

Question 9
1) The total momentum of an isolated system is conserved; alternately – in an isolated system, the total
linear momentum before a collision is equal to the total linear momentum after the collision
2) take forward (direction of minibus) as positive …
( mc vc  mt vt )i  ( mc vc  mt vt ) f  (1500  0)  (2000  20)  (1500 12)  2000v
 v  11 m  s -1
3) The driver will continue to move forward at the same velocity until s/he hits the windscreen and it brought
to a stop. This is the result of the inertia of the driver.
st
4) Inertia – Newton’s 1 Law of Motion - An object continues in a state of rest or uniform motion (motion with
a constant velocity) unless it is acted on by an unbalanced (net or resultant) force.

Question 10
1) to the left (in opposite direction to skateboard motion)
2) Newton’s Third Law
3) In a closed system, and in the absence of dissipative forces, the total amount of mechanical energy
remains constant.
( E p  Ek ) A  ( E p  Ek ) B  (70  4)(9, 8)(0)  12 (66) v  66  9, 8  1, 6  12  66  0
2

4)
v  66  9, 8  1, 6 / 33  5, 6 m  s
-1

5) Total momentum before = total momentum after, therefore


70  5  66  5, 6  4v parcel  v p  4, 9 m  s -1
-1
- the velocity of the parcel is 4,9 m·s to the left

6) If the parcel is thrown to the left with a greater velocity, the velocity of the boy, immediately after release of
the parcel, will increase. Having a velocity greater than the minium required to reach the top of the slope
(as calculated in 4)), the boy will now have a positive kinetic energy at the top.

Question 11
1) Momentum is the product of an object’s mass and its velocity.
2) take the direction of motion as positive ..
-1
Then Δp = m(vf – vi) = 175(0 – (+20)) = – 3500 kg·m·s
pg 8
-1
The change in momentum is 3500 kg·m·s in a direction opposite to motion.
3) take direction of motion as positive
FnetΔt = Δp, thus f×8 = –3500, thus f = –437,5 N, i.e. opposite to the direction of motion.

Question 12
1) An elastic collision is one in which both total momentum and total kinetic energy are conserved.
2) for ball A: EM-top = EM-bottom,
2 2
thus (½ mv + mgh)top = ( ½ mv + mgh)bottom
2
thus ½ × 0,2 × 0 + 0,2 × 9,8 × 0,2 = Ek + m × 9,8 × 0
thus Ek = 0,39 J
2
3) Total Ek before = total Ek after, thus 0,39 + 0 = ½ × 0,2 × vf + 0,12
-1
Thus vf = 1,64 m·s .
2 2
4) Ek before = ½ mA viA , thus 0,39 = ½ (0,2)viA
-1
viA = 1,98 m·s
Impulse = m(vf – vi) = 0,2(-1,64 – 1,98) = – 0,72 N·s
The magnitude of the impulse is 0,72 N·s

Question 13

1) Total momentum before = total momentum after


-1
Thus (5)(4) + (3)(0) = (5+3)vf, thus vf = 2,5 m·s .
2) FnetΔt = m(vf – vi), thus Fnet(0,3) = (8)(0 – 2,5)
thus Fnet = – 66,67 N
the magnitude of the net force acting on the combined blocks is 66,67 N.

pg 9
pg 10
Grade 12
Exam Preparation

Mechanics III
Memorandum

TouchTutor® Series
Physical Sciences

Section A (Multiple Choice Questions)


2 2 2 2
1) B – W net = ∆Ek = ½m(2v) - ½mv = 3(½mv ) = 1½ mv
2) A – the law of the conservation of mechanical energy
2 2
3) C - work done initially: W net = ∆Ek = ½mv - 0 = ½mv = W
nd 2 2 2
work done in 2 part: W net = ∆Ek = ½m(2v) - ½mv = 3(½mv ) = 3W
4) C – power is the rate at which work is done (or energy is used): P = W/t
5) D – the weight of the car pulls the car down, working in the direction opposite to motion, i.e. negative work
is done by the weight of the car. Since it moves up the hill at constant speed, there is no change in the
kinetic energy of the car. Only its potential energy changes. The car’s motor must do positive work to
overcome the negative work of the car’s weight.
6) B – Wnet = ∆Ek, so if the net work is zero, there cannot be a change in kinetic energy, thus the object
moves with constant speed.
7) D –
8) B – W nc = ∆Ek +∆Ep – which is equal to the change in mechanical energy of an object
9) C – the force acting in the direction of the motion does positive work
2 2
10) D – 8E. Ek (of Y) = ½ (2m)(2v) = ½ ·2·4 mv = 8E
11) A – equal momentum: m1v1 = m2v2, but m2 = 4m1, therefore v2 = v1/4.
2 2 2 2
Ek1 = ½ m1v1 ; Ek2 = ½ m2v2 = ½ (4m1)(v1/4) = (4/16) × ½ m1v1 = Ek1/4 – the ratio is 4:1
12) D – the girl does the same amount of work as the boy, but in less time. She has more power (work per

unit time)
13) B – 64,8 m. A person expends 75 W of power of 90 seconds, and since P = W/t, the person does 75 × 90
= 6750 J of work. This work is done by the force (or component of the force) in the direction of motion.
The component of F in the direction of motion = F cos 25° = 104,23 N. Since W = F∆x, we know that ∆x =
W/F = 6750 / 104,23 = 64,76 m.
14) B – statements (i) and (ii) are true. (iii) is not true – the work done is equal to the change in kinetic energy.
15) C – there is a resultant force to the left, thus acceleration to the left.
16) B – 600 J. At the lowest point, all the potential energy has been converted to kinetic energy. This then is
the total energy the system has.
17) C – W = F·∆x = F·(cos θ)·D = Fdcosθ
2 2
18) C – the work done in bring car to rest = W net = ∆Ek = 0 – ½ mv = – ½ mv
This work is done by the force of friction = μN = μmg,
2
and since W = F·d, – ½ mv = – μmg·d
2
thus d = mv / 2mμg , and m cancels above and below the line
therefore the distance d skidded is independent of the mass of the car – a car with mass 2m will
skid as far.
2
19) D – it will now travel four times as far. From the previous question: d = v / 2μg, thus if v is doubled:
2 2
dnew = (2v) / 2μg = 4 v / 2μg = 4d
20) B – it turns into thermal energy and then is dissipated (spread to the environment)

pg 11
Section B (Structured Questions)
Question 1
1) Given that friction (a dissipative force) is involved in moving down the incline,
the total mechanical energy is not conserved.
2) Conservative force: one for which work done by or against it depends only on
the start and end points of a motion and not on the path taken.
3) Note: the vertical downward force due to gravity is replaced by its two
components (C – parallel to incline and D – perpendicular to incline)
4) C is the component of Fg parallel to incline …
1, 5
Fg ||  mg  sin( )  120  9, 8   176, 4 N
10
5) 0 J. Force D acts perpendicular (at right angles) to the direction of motion, and thus no work is done by
force D.
6) The work done on an object by a net force equals the change in kinetic energy of the object – W net = ΔEk
7) a) B is the frictional force – note it acts to oppose motion (in opposite direction to motion).
Wf  Ff  x  cos( )  50  10  cos(180)  500 J
b) C is the parallel component of the weight, pulling the wheelchair down the incline ..

WF  Fg ||  x  cos( )  176, 4  10  cos(0)  1764 J


g ||

8) You can ignore forces A and D – since they’re perpendicular to the motion, they do no work. The
wheelchair moves down the incline with constant velocity, hence ΔE k = o J
Wnet  Ek WB  WC  WE  Ek  (500)  (1764)  WE  0 WE  1264 J
The health care worker must take energy out of the system to prevent the wheelchair accelerating down
the incline.
9) The appropriate power formula is P = Fv. The velocity of the wheelchair is given. Calculate force exerted
by worker using the work calculated in previous question.
WE  FE  x  cos( )  1264  FE  10  cos(180) FE  126, 4 N
P  Fv  126, 4  1, 25  158 W

Question 2
1) In a closed system, and in the absence of dissipative forces, the total
amount of mechanical energy remains constant.
2) a) and b)
3) Total mechanical energy (Ep + Ek) at P is equal to total mechanical
energy at R, therefore …

Ek  P  E p  P  E k  R  E p  R  0  (40)(9, 8)(1, 5)  12 (40)v  0


2

2
 v  29, 4  v  5, 422 m  s
-1

4) There are two forces acting on the girl – gravity (use parallel comp.) and friction ..
Wnet  Ek  Fg ||  x  cos( )  Ff  x  cos( )  12 m(v f  vi )
2 2

 (22)(9, 8) sin(30)(3) cos(0)  (1, 9)(3) cos(180)  12 (22)(v f  0)  v f  5, 374 m  s


2 -1

5) With no friction acting, the girl would have the same final velocity as the boy – just leave out the frictional
force in the previous calculation to confirm this.

Question 3
1) Power is the rate at which work is done.
2) Power = W/Δt (and since work = energy) = ΔE/Δt
7
then ΔE = 88 000 × (7 × 60) = 3,696 × 10 J
7
3) Ep = mgh = (3000)(9,8)(1066 – 366) = 2,058 × 10 J. This value is lower than that calculated in 2)
because some energy is lost in overcoming friction. Energy is also required to acceleration the cable car
initially.
4) Assume an isolated system. The total mechanical energy at the top (potential energy only since car is at

pg 12
rest) minus the energy lost due to friction = total mechanical energy at bottom (kinetic energy).
EM top  E friction  EM  bottom  mgh  E f  12 mv  (3000)(9, 8)(1066  366)  6  10  12 (3000)v f
2 6 2

 v f  9720  v f  98, 59 m  s
2 -1

Question 4
1) Gravitational potential energy is the energy a body possesses due to its position above a particular
reference point (particularly the Earth’s surface)
4 8
2) Ep = mgh = 6×10 × 9,8 × 948 = 5,574 × 10 J
3) a) Kinetic energy is converted into mechanical and then electrical energy
8 6
b) P = W/t = ΔEp / t = 5,574 × 10 / 60 = 9,29 × 10 W. (convert minutes into seconds)
6 6
however, there is only a 80% conversion, hence power output = 0,8 × 9,29 × 10 = 7,432 × 10 W.
c) Assuming the conservation of mechanical energy, the potential energy of 1 kg of water at the top will be
converted into kinetic energy at the bottom, therefore
E p  mgh  (1)(9, 8)(948)  9290, 4 J - the average kinetic energy then = 92940,4 J.

Question 5

3 -1
1) P = F·v, therefore 300 = 6,8×10 · v, ∴ v = 0,04412 m·s . time = distance /
velocity = 15 / 0,04412 = 339,98 s or 5,67 min (or W = Fd = 102 000J; t = W/p
= 102000/300 = 340 s)
2) See graph to the right … Work = force × distance = area under the graph = ½
(15)(210 – 40) + (15 × 40) = 1875 J
3) No, work is purely a function of force and distance, thus how fast you raise the
book makes no difference to the amount of work done. Power is the rate at
which work is done, so clearly the faster your raise the book, the more power is
delivered.
4) Let’s assume the elevator lifts the mass at constant velocity – to do so, it must
counter the force of gravity on the mass = mg.
3
Now P = W/Δt = (F·Δy)/Δt = mgΔy/Δt = (1,1 × 10 )(9,8)(40) / 12,5 = 34496 or 3,4
4
× 10 W
5) W = F·Δx = mgΔx, therefore 176 = m(9,8)(0,3), therefore m = 59,864 or 59,9 kg
3 7
6) W = F·Δx = (551)(161 × 10 ) = 8,87 × 10 J
7) a) see graph to right
b) W = F1d1 + F2d2 + F3d3 = (20)(5) + (35)(12) + (10)(8) = 600 J

Question 6
1) To calculate the magnitude of the frictional force we must first calculate the normal force

N  mg cos(25)  (300)(9, 8) cos(25 )  2664, 545 N


Ff   k N  0,19  2664, 545  506, 26 N
2) There are 3 forces to consider – the motor’s force applied (Fapp), the force of friction (Ff) and the
component of the force of gravity parallel to the incline (F g||) – the last two in opposite direction to the first.
Since the crate moves up the incline at constant speed, the resultant force is zero.
Fres  Fapp  Ff  Fg ||  0  Fapp  ( 506, 26)  ( 300  9, 8  sin(25))  0  Fapp  1748, 76 N
Pave  Fvave  1748, 76  0, 5  874, 38 W

Question 7
1) Note: the depth of the well is measured from point A.
Pumping the water out, it is given potential energy and kinetic energy … work is done by the pump
2 2 2
Ep = mgh = (90)(9,8)(37) = 32634 J Ek = ½ mv = ½ (90)(2 – 0 ) = 180 J
The pump thus must supply 32634 + 180 = 32814 J of energy.
2) P = W/t = 32814 / 60 = 546,9 W
3) a) o,5 kW = 500 W which is less than the required 546,9 W – thus the petrol pump will not be able to
pump the water out at the same rate.
b) A windmill has some advantages – it is environmentally friendly and except for maintenance costs, is

pg 13
cost-free. It will however not work when there is no or very little wind. A petrol pump (stronger than
0,5 kW) would be more efficient and will work as long as petrol is available. The petrol does however
cost, and the price of petrol is constantly increasing.

Question 8
1) Assume that all the kinetic energy of the bullet is transferred to the bullet-block system, with no energy lost
due to the deformation of the block. Immediately after collision, the only force acting on the system is the
frictional force of 10 N.
Wnet  Ek  f  x  cos(180)  12 (m)(v f 2  vi 2 )
 (10)(2)(1)  12 (7,02)(0  v 2 )  v  2,39 m  s -1
2) The total momentum of an isolated system is conserved; alternately – in an isolated system, the total
linear momentum before a collision is equal to the total linear momentum after the collision
3) total momentum before = total momentum afterwards …
mb vb  mblock vblock  mbullet block v  (0,02)vb  0  (7,02)(2,39)
 vb  838,89 m  s -1
-1
The bullet travels at 838,89 m·s towards the block

Question 9

1) EM ( top )  EM P  mgh  12 mv 2  (5)(9,8)(5)  12 (5)v 2  v  9,90 m  s-1


2) PQ is a horizontal, frictionless surface, therefore no energy is lost to friction and none is gained or lost as
potential energy. Consequently the kinetic energy will be the same at P and Q.
3) A non-conservative force is any force for which the work done is path-dependent.
4) To calculate θ, we need to calculate the length of QR - use the work-energy theorem for this.
Wnc  Ek  E p  f  x  cos(180)  12 m(v 2f  vi2 )  mg (h f  hi )
 18  x  12 (5)(9,902 )  (5)(9,8)(3) x  5, 4458 m
Now sin( )  3 / 5, 4458  0,551   sin 1 (0,551)  33, 43

Question 10
1) see diagram to right
Wnet  Ek  WF  WF  W f  Ek
g
(W  F  y  cos  )

2) 17000  20  cos(0)  (1600  80)(9, 8)  20  cos(180)  W f  0 (const. velocity)


 W f  10720 J

pg 14
Grade 12
Exam Preparation

Waves, Sound
and Light
Memorandum

TouchTutor® Series
Physical Sciences

Section A (Multiple Choice Questions)


1) D – if source and listener travel towards each other, the frequency heard will be higher. To answer
Doppler effect questions, always draw a diagram.
2) A – approaching, the heard frequency will be constant
and higher, and leaving the listener, the frequency will
be constant and lower.
3) C – the Doppler effect can be observed in all three types of waves.
4) B – 1 kHz (or 1000 Hz), i.e. the same frequency. Since there is no relative motion between the two
vehicles, no Doppler effect will be perceived.
v  vL 340
5) A – with the train approaching, fL   fS   2000  2222,22 Hz
v  vS 340  34
v  vL 340
with the train receding: fL   fS   2000  1818,18 Hz
v  vS 340  34
the change in frequency = 2222,22 – 1818,18 = 404 Hz
6) C – according to the Doppler effect, the frequency heard will be higher or lower than the frequency of the
source when source and listener are moving relative to each other. There is no relative movement here,
so the answer is C. The wind direction has no significant effect on the frequency of the sound.

7) D – the heard frequency will change in a step-like manner, and will decrease.
8) Redshift is a shift of the light frequency towards the red end of the spectrum, i.e. a lower frequency and
energy (E = hf) and a higher wavelength – thus the correct answer is B.
9) D – the spectral lines are clearly redshifted, i.e. the star is moving away from the Earth.
10) C – destructive interference occurs when a crest meets a trough, i.e. when the displacement of the wave is
in opposite directions.
11) C – superposition
12) C – a wavefront. This is an example of Huygen’s principle.
13) B – between 1,5 and 3 s later. Think this through carefully and remember that the stone is
accelerating, thus covering the second half of the distance down the well (equal to the first
half) in less time (t3) than it takes to travel the first half (t1, with t3 < t1). Sound however
moves at a constant speed, and will cover the first half of the return journey (t4) in the same
time as the second half (t2). For the first drop, t1 + t2 = 1,5. Now t3 + t4 < 1,5, therefore total
time taken is between 1,5 and 3 seconds.
14) D – the speed of the wave changes, and since the frequency remains constant, the wavelength must
necessarily change. Remember v = f × λ
15) C – zero. Remember that a point on a wave never moves left or right (waves are a transfer of energy, not
matter). Point A, for a brief moment while at the top of the crest, does not move (i.e. has zero velocity). At
the very next moment though, point A would move downwards.

pg 15
c 3  108
16) B – c  f  ,    0,03 m  3 cm
f 10  109
17) B – the compression and rarefaction as parts of a wave travel in the same direction. Individual particles
may travel in the opposite direction for a short time.

Section B (Structured Questions)


Question 1
1) The car must be moving away from the observer. When the car moves away, the perceived frequency
(the freq. measured by the observer) will be lower than the actual frequency of 850 Hz and thus the
detector registers a reading. If the car were moving towards the observer, the perceived freq. would be
higher than 850 Hz, and then the detector would not register anything.
v  vL 340
f   fS  800   850  vS  21, 25 m  s
-1
2)
L
v  vS 340  vS
3) The Doppler effect is used to measure the rate of blood flow. It is also used to measure foetal heart beat.
Question 2
1) The perceived frequency changes because the observer drives past the stationary ambulance.

v  vL 340  vL
a) f   fS  900   850  vL  20 m  s
-1
2)
L
v  vS 340
b) v  d / t  20  d / 6  d  120 m - the car is 120 m away from the ambulance when the
observer begins to measure the frequency.
Question 3
1) Doppler effect: The apparent change in the frequency of a wave when there is relative motion between the
source of the wave and an observer.
2) a) The perceived pitch will be higher than the actual frequency emitted because the sound waves are
‘compressed’ in front of the plane. The ‘compression’ leads to a higher frequency. (Draw a diagram to
illustrate – a diagram will always help you clarify the Doppler effect)
b) Apply the Doppler effect equation for the two situations – as the
jet approaches, then as the jet leaves … the difference in the
perceived frequency before and after is 430 Hz.
v  vL 330 v  vL 330
before: f   fS   fS after: f   fS   fS
L
v  vS 330  245 L
v  vS 330  245
330 330
now:  fS   f S  430  3,882 f S  0, 574 f S  430
330  245 330  245

Therefore fs = 129,988 Hz
3) a) red shift
b) a shift of the visible light frequencies towards the red side of the spectrum implies a lengthening of the
wavelength, or lower frequency. This implies that the stars are moving away from us.
Question 4
1) the hawk is moving away from the bird watcher
2) Doppler effect
v  vL 343
f   fS  700   900  v  98 m  s
-1
3)
L
v  vS 343  vS
4) The speed of the listener / observer is kept constant, the direction of the sound source is also kept
constant. Another constant is the frequency of the sound source since it is the same radar gun (the
source of the waves reflected by the moving vehicles) that is used for each approaching vehicle.
5) the velocity of the sound source is the independent variable (plotted along the x-axis) while the frequency
heard by the listener is the dependent variable (plotted along the y-axis)
6) The original hypotheses should be rejected since the frequency heard increases, rather than decreases,
as the velocity of the sound source increases.

pg 16
Question 5
The further away the star or galaxy, the more its incoming light frequency is red-shifted. This implies that
the universe is expanding, and expanding at an ever-increasing rate, i.e. the farther away the star or
galaxy is, the faster it is moving away from us.
Question 6
1) Spectrum B is red-shifted relative to the standard reference A, while spectrum C is blue-shifted.
2) The light source for B is moving away from the observer, while the light source for C is moving towards the
observer – this is the Doppler effect applied to light waves.
Question 7
1) Doppler effect
2) Because the car is travelling away from the observer, the frequency heard will be below the frequency of
the source, 265 Hz. As the car slows down, the pitch (or the perceived freq) will gradually get closer to the
true pitch of the car, i.e. the heard frequency will rise.
3) The speed of sound is much lower than the speed of light. The Doppler effect is only noticeable if the
speed of the source is significant compared with the speed of the wave. Hence the effect is noticeable with
sound but not with light. By way of comparison:
vsource 15 vsource 15
  4, 4 %   5  106 %
vsound 340 vlight 3  10 8

v  vL 340
4) f   fS   265  277, 23 Hz
L
v  vS 340  15
Question 8
1) The apparent change in the detected frequency as a result of the relative motion between a source and an
observer (listener).
v  vL 340  (340  310)
2) f   fS   280  304, 71 Hz
L
v  vS 340
3) Smaller than - if the listener moves away from the source (rather than towards the source) at constant
velocity, the frequency detected will be lower than the frequency emitted by the source.
4) The Doppler flow meter can be used to determine the rate of blood flow / it can be used to monitor the
hearbeat of a foetus.
5) The wavelength appears to be increasing, which implies that the star is moving away from the earth.
Question 9
1) Formulate an equation for each of the two situations – the train approaching (A) and the train moving away
(B). The frequency of the whistle is the same in both cases. We change the subject of the formula to
make for easier understanding of the problem. vtr is the velocity of the train, ftr is the frequency of the
whistle.

v  vL f L  (v  vtr ) 590  (343  vtr )


A: f   f tr  f tr  
L
v  vtr v 343
v  vL f L  (v  vtr ) 540  (343  vtr )
B: f   f tr  f tr  
v  vtr
L
v 343
590  (343  vtr ) 540  (343  vtr )
 
343 343
 590  (343  vtr )  540  (343  vtr )  vtr  15,177 m  s -1
-1
The train travels at a constant 15,18 m·s , all the while blowing its whistle.
2) Having calculated the velocity of the train, and being given the frequency the observer hears, we can now
calculate the frequency of the whistle. Take situation A – the train approaching … in terms of the Doppler
equation, the velocity of the train is then subtracted …
v  vL v  vS 343  15,177
f   fS  fS   fL   590  563,894 Hz
L
v  vS v  vL 343  0
Repeat this calculation, but now for situation B – the train moving away from the observer to confirm the
frequency of the whistle. It should be exactly the same … (note, the velocity of the train is now added …

pg 17
v  vL v  vS 343  15,177
f   fS  fS   fL   540  563,894 Hz
L
v  vS v  vL 343  0
Note: to determine the true frequency of the whistle, you cannot just take the average of the frequencies
heard before and after the train passes. This would give you 565 Hz, a close enough approximation, but
not the correct answer.

pg 18
Grade 12
Exam Preparation

Electricity and
Magnetism I
Memorandum

TouchTutor® Series
Physical Sciences

Section A (Multiple Choice Questions)


1) A – applying Fleming’s left hand rule
2) D – there is no such thing as a single magnetic pole – any magnetic object will always, always have both a
north and a south pole.
3) C – the direction of an electric field is the direction in which a positive test charge would move if placed in
the field – thus from positive to negative
4) C – the total resistance decreases with the addition of a resistor in parallel and so the total current
increases
5) B – the electrostatic force between two charged sphere α product of their charges. If one charge is
halved, all else being the same, the force will be halved.
6) A – E = F/q – the force per unit positive charge at that point.
2
7) A – 3 N. If the distance is doubled, the force is reduced to ¼ of the original force (F α 1/r )
8) B – the forces from Q1 and Q2 must be equal and cancel, which they do since Q2 = -4Q1.
2
F1 on 3 = kQ1Q3/a .
2 2 2
F2 on 3 = kQ2Q3/x = k(4Q1)Q3/x = 4kQ1Q3/x .
Now kQ1Q3/a = 4kQ1Q3/x , therefore 1⁄ 2 = 4⁄ 2 ∴ 𝑥 2 = 4𝑎2 ∴ 𝑥 = 2𝑎
2 2
𝑎 𝑥
9 -6 -12 2 -8
9) A – Fq1 on q3: (9×10 × 8,1×10 × 2,16×10 )/2,62 = 2,29 × 10 N to the left (attraction)
9 -6 -12 2 -8
Fq2 on q3: (9×10 × 2,16×10 × 2,16×10 )/0,91 = 5,07 × 10 N to the right (repulsion)

-8
Magnitude of the total force on q3 = 2,77 × 10 N
10) A – the voltmeter must be attached across the resistor (i.e. in parallel), while the ammeter must measure
all the current going through X.
11) A – when the switch is closed, the resistor r will be bypassed (not current will flow through it). The
voltmeter will measure the potential difference across a length of the conducting lead, and that,
theoretically at least, is zero.
12) D – charge. The formula for charge is Q = It (ampere second)
13) C – resistor in parallel are current dividers / they have the same potential difference across them. The
larger resistor will carry less current than the smaller resistor.
2
14) C – 90 watts. V = PR = 10(3R) (for the series circuit) and P(R/3) (for the parallel circuit); therefore P (for
the parallel circuit) = 30R/(R/3) = 90R/R = 90 W.
15) C – the voltage between P and Q is 22,5 V. RPQ = 4,4 Ω, and RQR = 3,4 Ω. The voltage is split in the
same ratio, i.e. the voltage PQ = 4,4/(4,4+3,4) × 40 = 22,5 V.
16) C – P = VI = 20 × 0,8 = 16 W
17) B – the internal resistance of a battery or cell is an important factor when the external circuit has a very low
resistance (or a resistance of relative size to the internal resistance). When the ext. resistance is very
high, the internal resistance of the battery is much less significant. Low current loads generally imply a
high external resistance.
18) B – study the circuit carefully to confirm that this is the correct answer.

pg 19
19) A – 1,5 A. The 2A current (moving right down) is split into two, the 1A current
shown, and 1A flowing from junction toward the next junction. That this latter
junction, the 1 A current and the 0,5 A current combine, thus I = 1,5 A.
20) A.

Section B: Structured Questions


Question 1
1) Coulomb's law states that the magnitude of the electrostatic force between two point
charges is directly proportional to the product of the magnitudes of the charges and
inversely proportional to the square of the distance between them …
2) see diagram to right
3) Calculate each of the forces, then use Pythagoras’ theorem to calculate the magnitude and direction of the
resultant force.
(note: in calculations involving positive and negative charges, first calculate the magnitude of the forces,
etc. involved, then decide on the direction depending on whether these are attractive or repulsive forces)
6 6
Q1Q2 (4  10 )(4  10 )
F (Q2 on Q1): F  k  (9  10 )  1, 6  10 N (left)
9 4
3
(3  10 )
2 2
r

6 6
Q1Q2 (4  10 )(4  10 )
F (Q3 on Q1): F  k  (9  10 )  1, 6  10 N (down)
9 4
3
(3  10 )
2 2
r

(1, 6  10 )  (1, 6  10 )  2, 263  10 N


4 2 4 2 4
Magnitude of resultant force:

1, 6  10
4
FQ 2
Direction: tan( )   1    tan (1)  45
1

1, 6  10
4
FQ 3
4
The resultant force is 2,263 × 10 N, with a bearing of 225°

Question 2
1) a) emf  I ( R  r )  6  I (9  1)  I  0, 6 A
b) First calculate the resistance of R1 based on the power dissipated.
P  I R 1, 8  (0, 6) R  R  5 
2 2
- then calculate R2, given that the total external resistance = 9
Ω and R3 = 4 R2. Rp  9  5  4 

1 1 1 1 1 1
      R2  5 
RP R3 R2 4 4 R2 R2

W  VI t  (240)(9, 5)(12  60)  1, 642  10 J


6
2)
The kilowatt-hour (symbolized kWh) is a unit of energy equivalent to one kilowatt (1 kW) of power
expended for one hour. One watt is equal to 1 J/s. One kilowatt-hour is 3,6 megajoules.

kW  h  (3600 s)[kW]  3600 s 


 kJ   3600 kJ  3, 6 MJ
 s 
Thus energy used = 1, 642  10 / 3, 6  10 kWh . This, multiplied by the cost per kWh (R 1,47) = R 0,67
6 6

Question 3
1) Coulomb’s law
2) The three charges are equal and positive … while the field lines to the left of A and between A and B are
ok, there are a number of issues with the drawing …
(i) field lines at B cross each other – field lines should never cross
(ii) the field lines around C should have the same density as those around A and B
(iii) the field lines around C should move away from C, not towards C
3) A point charge at C will experience a force of repulsion due to B and a force of repulsion due to A.

pg 20
6 6
Q1Q2 (100  10 )(100  10 )
FBC  k  (9  10 )  1  10 N (right)
9 5

(3  10 )
2 2 2
r
6 6
Q1Q2 (100  10 )(100  10 )
FAC  k  (9  10 )  2, 5  10 N (right)
9 4

(6  10 )
2 2 2
r

Total force experienced by point charge at C = 1  10  2, 5  10  1, 25  10 N to the right


5 4 5

4) The force on a point charge at B due to A will have the same magnitude but opposite direction to the force
on B due to C, therefore the total force experienced at B will be 0 N.

Question 4
1) As the spindle turns, it turns a magnet – this produces a changing magnetic flux through the coil just below
it, and thus induces a potential difference measured by the voltmeter.
2) Faraday’s law of electromagnetic induction (see above)
3) use a more powerful magnet / add more turns to the coil / ensure that
any friction preventing rotation of the spindle is minimized / use larger
but lighter cups / minimize any weight that must be rotated
4) see diagram
5) a) hypothesis: as the cup size increases, the voltage increases

b) independent variable: size of cups; dependent variable – the induced voltage


c) the same magnet should be used, with the same number of turns in the coil, the number of cups
should be constant, as well as wind speed and wind-direction. The same iron nail should be used.

Question 5
for X : P  VI  I  P / V  36 / 12  3 A
1)
for Y : P  VI  I  P / V  2, 0 / 4, 5  0, 44 A
2) The voltage across lamp X is 12 V, thus the voltage across the R 1 resistor must be 24 – 12 = 12 V
(resistors, lamps etc. in series are voltage dividers)
3) Total current through the circuit = current through the two parallel branches (parallel branches are current
dividers) = 3 + 0,44 = 3,44 A
4) R1 = V / I = 12 / 3,44 = 3,488 Ω
5) Potential difference across the R2 resistor = 12 V (the potential difference across the whole parallel
combination) – 4,5 V (the voltage across Y) = 7,5 V. R = V/I = 7,5 / 0,44 = 17,05 Ω.
6) When lamp X stops conducting electricity, the total resistance of the circuit increases, thus the current
through the circuit will decrease. Since R1 remains constant, and V = IR, if the current decreases, then the
voltage must also decrease.
7) When lamp X stops working, the resistance of the parallel combo (now just R2 + Y) increases. With a
2
greater resistance comes a greater portion of the total 24 V potential difference. And since P = V /R, a
greater voltage must lead to greater power and greater brightness for lamp Y.

Illustrated through calculation:


2 2 2
Lamp Y: P = V /R, therefore R = V /P = (4,5) / 2 = 10,125 Ω
Initial voltage division between R1 and (R2 + Y / X) was 12 : 12
With R1 = 3,488 Ω; R2 + Y = 17,05 + 10,125 = 27,175 Ω – the 24 volts divided in ratio 2,73 : 21,27

Question 6
6 6 6 6
(9  10 )(2  10 )(3  10 ) (9  10 )(4  10 )(3  10 )
9 9

1) F1  2
 21, 6 N (left ) F2  2
 120 N ( right )
0, 05 0, 03
Total force: 120 – 21,6 = 98,4 N (to the right)
2) a) use Coulomb’s law to calculate both forces on the middle particle …
answer: 17,96875 N right
b) answer: 42,3177 N to the right.

(9  10 ) X
9 2
Q1Q2 7
3) F k  0, 28  2
 X  6, 693  10 C
(12  10 )
2 2
r
4) Here is a force diagram of the situation …
3 3
a) Fg  mg  (1  10 )(9, 8)  9, 8  10 N

pg 21
b) Since the forces on the suspended ball are in equilibrium, F E = the horizontal component of T, Tx, and
Fg = vertical component of T, Ty
-3 -2
Therefore Ty = T sin (60°) = 9,8 × 10 , therefore T = 1,13 × 10 N
-3
then Tx = T cos (60°) = FE = 5,66 × 10 N
c) The force between the two charged spheres = F E, therefore their charge, (being equal) =

(9  10 ) X
9 2
Q1Q2 3 8
F k  5, 66  10  2
 X  2, 379  10 C
(3  10 )
2 2
r
8
E  F / q  0, 06 / (2  10 )  3  10 N  C
6 -1
5) - with direction to the right.

Question 7
2 2
1) the 50 W bulb. P = V / R, therefore R = V / P, thus the lower P is, the higher R must be.
2) The resistance of the wire and the current passing through the wire should be as low as possible.
3) a) I = V / R = 27 / 18 = 1,5 A b) 27 V c) P = VI = 27 × 1,5 = 40,5 W
5
d) E = P·t = 40,5 × 3600 = 1,46 × 10 J
4) a) No. The voltage is increased by a factor of 9/6 = 1,5, but the current is increased by a factor of 75/66 =
1,1 – thus the lamp does not obey Ohm’s law.
-3
b) P = V × I = 66×10 × 6 = 0,40 W.

c) P = VI = 75×10-3 × 9 = 0,68 W
5) The voltage is divided in the same ratio as the ratio of the resistors. Thus voltage across the 235 kΩ
235
resistor =  45  14, 894 or 15 V
235  475
6) a) 0 A – A and B are at the same potential and thus no current flows through the wire connecting A and B.
b) nothing – the current through each of the four resistors will remain the same
c) nothing – it remains the same
d) nothing – it remains the same
7) In a series circuit, the current is opposed by each resistance in turn. The total resistance is the sum of the
resistors. In a parallel circuit, each resistance provides an alternative path for current to flow along. The
result is a decrease in total resistance.

Question 8
1) a) 1,5 V (emf is measured when there is no current flowing through the external circuit)
gradient of line = I / V  0  1, 8 / 1, 5  0  1, 20
b)
r  V / I  1 / 1, 2  0, 83 
2) a) increases
b) emf = I(R+r) = IR + Ir = V + Ir. When the resistance of the rheostat is increased, and the emf of the

battery remains the same (as it should), the total current flowing is reduced, thus IR = V (across the
rheostat is reduced) increases.
3) a) 4 × 1,5 = 5 V.
b) (i) V  IR  I  V / R  2 / 4  0, 5 A
emf  I ( R  r )  0, 5  [( R p  4)  4(0, 25)]  6 V  Rp  7 
(ii) 1 1 1 1 1 1
      Rx  14 
RP R1 R2 7 14 Rx

pg 22
Grade 12
Exam Preparation

Electricity and
Magnetism II
Memorandum

TouchTutor® Series
Physical Sciences

Section A (Multiple Choice Questions)


1) C is a graph for rectified current (i.e. DC). D is a wave form, but not for AC or DC current. That leaves A
and B as possible answers. The correct answer is A – maximum voltage is produced when the coil is in
line with the magnetic field, cutting the magnetic field lines at right angles when it moves.
2) C – the coil with rotate anti-clockwise. Use Fleming’s left hand motor rule (it is a current-carrying
conductor).
3) A – at a power plant, mechanical energy (the turning turbine) produces electrical energy (AC current)
4) A – the frequency increases, since the polarity of the induced emf is changed more rapidly.
5) A – (1) is correct. (2) is not correct – the split ring commutator does not turn with the coil. (3) is not correct,
since the greatest force is experienced when the coil cuts the magnetic field at right angles (which
happens when the coil is lying in the same plane as the magnetic field.)
6) A
7) C – both ac and dc generators convert mechanical energy into electrical energy. Slip rings would indicate
an ac generator rather than a dc generator (split ring commutator). The same methods are used in both to
increase the induced voltage.
8) C – slip rings are used in AC motors and generators, but not in DC generators.
9) C – clockwise: use thumb in direction of current – curl of fingers gives direction of magnetic field
10) C – magnetic field lines always run from the north to the south pole.
11) B – to the top of the page .. apply Fleming’s left hand rule. 12) A.

Section B: Structured Questions


Question 1
1) a) A – coil, B – brushes, C – split ring commutator
b) to maintain contact between the external circuit and the coil
c) DC motor (identified by the split ring commutator)
d) The motor effect – this happens when the interaction between an external magnetic field and the
magnetic field around a current-carrying wire produces a force that rotates the coil
Vmax 1
2) a) Vrms    0, 707 V
2 2
b) The current period as depicted on the graph is 0,08 s. When the coil is rotated at twice the original
speed, the period will be halved, i.e. it will be 0,04 s.
2
c) Pave  Vrms I rms  (0, 707)( )  0, 99 or 1 W
2

Question 2
1) a) electrical energy is converted to mechanical energy in a motor
b) AC and DC motors differ in a number of respects …
pg 23
while DC motors generally make use of permanent magnets, AC motors often use electromagnets
DC motors use split rings to reverse current through the rotating coil while AC motors make use of slip
rings to maintain electrical contact / or alternately, reverse the direction of the magnetic field with a
stationary coil …
c) the wipers are run off the car battery which delivers direct current / direction of wipers must be
continuously reversed.
2) a) 1 – the magnets, 2 – the coil or armature
b) determine the direction of the coil using Fleming’s right hand rule (forefinger – N to S, middle-finger –
direction of current, thumb – direction of force): the coil turns anti-clockwise
c) Fleming’s right hand generator rule / right hand dynamo rule
d) see diagram – the degrees refer to the angle of the rotating coil from
the horizontal
e) in phase - maximum current goes with maximum voltage
f) Components 3 & 4 are slip rings and brushes that maintain electrical
contact between the external circuit and the rotating coil
g) change the AC generator to a DC generator by changing the slip rings,
replacing them with split rings.

Question 3
1) Slip rings
2) A: 0, 10, 20 ms B: 5, 15, 25 ms
3) At position A, with the coil parallel to the magnetic field, the change in magnetic flux (the coil cutting
through the magnetic field) will be a maximum. Emf is directly proportional to the rate of change of flux,
and thus emf is at a maximum in position A.
4) When rotated at half the original speed, the rate of change of flux is halved, hence the emf is halved.
Beyond that, the time taken for a full cycle is halved, thus the graph looks as follows …
5)

6) AC current can be easily transformed to a high voltage, low


ampere current for better longer range transmission

Question 4

1) a) Vrms  Vmax / 2  170 / 2  120, 208 V

b) I rms  I max / 2  0, 7 / 2  0, 495 A


Vrms Vmax / 2 Vmax
c) R    242, 857 
I rms I max / 2 I max

2) a) Vrms  Vmax / 2  425 / 2  300, 520 V

b) I rms  Vrms / R  300, 520 / 500  0, 601 A


3) Yes. What is important is the relative motion between the coil and the magnetic field, so either one can be
rotating.
4) Michael Faraday discovered that a voltage is induced in a length of electric wire moving in a magnetic
field. The induced voltage may be increased by using a stronger magnetic field, increasing the velocity of
the conductor, or increasing the effective length of the conductor.
5) The coil experiences the maximum turning force when it is parallel to the magnetic field so that rotation
produces the greatest change in magnetic flux. Similarly, the coil experiences the minimum turning force
when it is perpendicular to the magnetic field, with rotation producing the least change in magnetic flux.

pg 24
6) An electromagnet is a metal object that can be quickly magnetised due to current flowing through a coil
around the object.

Question 5
1) Electromagnetic induction - the production of an electromotive force across a conductor when it is exposed
to a time varying magnetic field.
2) increase the number of coils / increase the magnet strength / increase the speed of rotation
3) slip ring
4) a) rms (root mean square) of an AC voltage is the value of the voltage in DC that will produce the same
heating effect as the AC voltage. Mathematically, the RMS value is the square root of the mean (average)
value of the squared function of the instantaneous values.

b) Vrms  Vmax / 2  339, 45 / 2  240, 027 V

Question 6
2 2
V (340 / 2)
1) a) Pave  rms
100   R  578 
R R

340
b) Pave  Vrms I rms 100  I rms   I rms  0, 416 A
2
2) AC current can very easily be reduced in voltage (stepped down) to a voltage that is safe to use and loses
less energy.

Question 7
1) alternating current
2) 100 W – the power rating of the bulb (P = W/t). It uses 100 J of energy per second, converting it to heat
and light.
Vmax 311
3) Pave  V  I rms 
rms
 I rms 100   I rms  I rms  0, 45 A
2 2
4) AC can be stepped up (to a higher voltage, lower current) at the power station. This minimizes the loss of
energy during transmission. Transformers at end substations can then step down the voltage for use in
the home.

Question 8

1) increase the strength of the magnetic field / the number of coils / the speed
of rotation
2) a) The split ring commutator is replaced with slip rings.
b) If the generator rotates at twice the original speed, the amplitude of the
potential difference will double, as will the frequency. The graph should
then look as follows …
3) Remember: for appliances, the power output is always the average power,
Pave. Also, given alternating current, we use the rms values for voltage and
current, not maximum values.

Maximum voltage = 340 V, therefore Vrms = 340 / 2  240, 416 V


2 2
Vrms (240, 416)
Pave  120   R  481, 67 
R R

pg 25
pg 26
Grade 12
Exam Preparation

Matter &
Materials I
Memorandum

TouchTutor® Series
Physical Sciences

Section A (Multiple Choice Questions)


1) C – increasing the frequency of the light increases its energy, so the electrons ejected from the metal
2
surface travel faster (Ek = ½ mv ). However, the same number of electrons are ejected – this is a function
of the intensity of the light, which remains constant.
2) B – returning to the ground state from which it was excited means the same amount of energy is released.
Note, the electrons of an atom (or a particular electron) absorb or release the energy, rather than the atom
as a whole.
3) C – replacing a 200 W UV light with a 40 W UV light means the intensity of the UV light is decreased and
so the number of photo-electrons emitted decreases. The frequency, and consequently the energy, of the
UV light remains the same.
4) C – electrons are ejected from a metal surface by packets / quanta of light – by photons of light. Photons
of light behave like particles.
5) B – blue light. E = hf, and blue light has the highest frequency.
6) C – the work function (W 0) of a metal is the energy required to eject an electron from that metal, and that
energy = hf0, where f0 is called the threshold frequency (W 0 = hf0). The higher the threshold frequency,
the higher the work function of a metal.
7) C – decreasing the wavelength implies an increase in energy (E = hc/λ). Given a constant intensity, the
number of electrons emitted remains constant, but they will have more energy (be more energetic).
8) A – an increase in the intensity of the light (i.e. more photons of light per unit time) means the number of
photo-electrons ejected will increase in a given time. Since the energy of each photon is the same (the

frequency and wavelength remain constant), the photons still have the same energy and the emitted
electrons remain as energetic as before.
9) C – the threshold frequency is that frequency at which light ejects electrons from a particular metal.
10) D – E = hf, thus energy is directly proportional to frequency, and E = hc/λ, thus indirectly proportional to
wavelength.
11) A – a line absorption spectrum; the atoms / molecules in the cold gas absorb various frequencies of light –
the prism then disperses the light and what is seen on the screen is the visible light spectrum with black
lines where a frequency was absorbed by the cold gas.
12) D – each element and / or molecule absorbs or emits a very specific and unique set of light frequencies;
these can be used to identify the element or molecule
13) A – when falling back to a lower state, an electron emits a photon of light – it appears in the emission
spectrum
14) A – match the line in the mixtures spectrum with elements D and E …
15) A – a line emission spectrum
16) A – the black lines in an absorption spectrum for a
particular element will match the lines of an emission
spectrum (as shown in this diagram)
17) D – the degree of the diffraction of a wave is directly
proportional to the wavelength of the wave

pg 27
8
18) B – 𝑣 = 𝑓 × 𝜆 thus 𝑓 = 𝑣⁄𝜆 = 3 × 10 ⁄
16
= 2,44 × 10 .
1,26 × 10−8
19) A – a photon is an energy packet and thus has no mass and thus zero momentum (p = mv)
20) E – light behaves both as a wave (interference patterns) and as a particle (photo-electric effect)

Section B (Structured Questions)


Question 1
1) a) The photoelectric effect is the observation that many metals emit electrons when light shines upon
them. Electrons emitted in this manner can be called photoelectrons
b) The energy of a photon (E) = hf, and since f = c/λ, E = hc/λ.
34
(6, 63  10 )(3  10 )
8
hc 19
E   7, 65  10 J

9
260  10
-9
Since 1,8 × 10 J of energy are transferred per second, this implies that the number of electrons
9 19
released (1 photon releases 1 electron) = 1, 8  10 / 7, 65  10  2, 353  10 electrons
9

19 10
q  ne  qe  2, 353  10  1, 6  10  3, 765  10
9
C
c)
10 10
q  I t  I  q / t  3, 765  10 / 1  3, 76  10 A

2) a) a line emission spectrum is formed by the frequencies of the electrons that, having absorbed energy
from an incoming energy source (e.g. a photon) – being in an excited state, release energy in specific
frequencies (based on the energy levels of the material’s atoms) as they fall back to lower energy levels.
b) line emission spectra are used to identify elements, and are especially useful in astronomy to analyse
the light that comes from stars

Question 2
1) a) kinetic energy b) frequency c) the type of metal use
2) For a given metal, there exists a certain minimum frequency of incident radiation below which no
photoelectrons are emitted. This frequency is called the threshold frequency.
14
3) 9 × 10 Hz
34 34
E  W0  Ek  hf  hf 0  Ek  (6, 63  10 )(14  10 )  (6, 63  10 )(9  10 )  Ek
14 14

4)
19
 Ek  3, 315  10 J
5) The kinetic energy will remain the same since changing the intensity of the incident light only changes the
number of photons striking the metal surface but not their frequency / energy.

Question 3
1) a) photo electrons b) electrons

2) photons
3) a) the reading will increase
b) greater intensity implies a greater number of photo electrons released per second. This implies greater
current.

Question 4
1) UV light removes (photo) electrons from the zinc plate and thus the leaves in Q will drop.
2) & 3) the UV light will again remove electrons which means that the leaves will be positively charged, and
the repulsion of like charges will cause the leaves to separate.
4) The frequency of white light is lower than the frequency of UV light.
34
(6, 63  10 )(3  10 )
8
hc 18
5) E   1, 989  10 J

9
100  10
18 19 18
E  W0  Ek 1, 989  10  6, 9  10  Ek  Ek  1, 299  10 J
18 31
Ek  12 mv 1, 299  10  12 (9,11  10 )v  v  1, 689  10 m  s
2 2 6 -1
6)

pg 28
Question 5
1) continuous spectra, line emission spectra and line absorption spectra.
Continuous – light emitted by the sun, or a star, is generally a continuous spectrum
Line emission – formed when an element or compound is heated and then emits light of specific
frequencies which are seen as lines of different colours / frequencies.
Line absorption – formed when the elements / compounds of a gas absorb certain frequencies of white
light shone through it …
2) False – the lines are for exactly the same wavelength or frequency, the only difference being that in
emission spectra, the lines appear as coloured lines against a darker background, while in an absorption
spectrum, the lines are black against a continuous colour spectrum.
3) Yes – the energy levels determine what quantities of energy (and E = hf) are released as an electron falls
to a lower energy level.
4) Each element has a different configuration of electrons and energy levels. This accounts for the difference
in line spectra.
5) No. As white light passes through the Earth’s atmosphere, certain energies would be absorbed by the
gases composing the atmosphere. Its spectrum, therefore, would have black lines on it.
6) Six lines are possible, corresponding to the various differences in energy levels. The drop E4 to E1 would
produce photons with the highest energy.

c  f   f  c /   3  10 / 513  10  5, 848  10 Hz
8 9 14
7)

Question 6
19 34
a) W0  hf0  2, 7  1, 6  10  6, 63  10  f0  f0  6, 516  10 Hz
14
1)

b) c  f      3  10 / 6, 516  10  4, 604  10 m or 460, 4 nm


8 14 7

34
(6, 63  10 )(3  10 )
8
hc 19
2) E   9, 945  10 J

9
200  10
19 19 19
E  W0  Ek  9, 945  10  2, 7  1, 6  10  Ek  Ek  5, 625  10 J
3) a) a photon
-34
b) 6,63 × 10 J·s – E = hf
c) blue light, since it has a higher frequency than red light
d) see above
e) because the frequency of the violet light exceeds the threshold frequency of the surface, while the
frequency of red light does not.
f) Yes – bright light implies a higher intensity, and thus more photons striking the surface ejecting photo
electrons
g) it supports the particle model of light – it shows that light comes as quanta (or packets) of energy
h) the photons / quanta of energy

i) the energy of a photon of light released is equal to one of the differences in energy levels of the atom
from which it is released
4) To eject electrons, the frequency of the incident photons must be higher than the threshold frequency.

Question 7
1) The work function of a particular metal is the minimum energy needed to emit an electron from the metal.
The frequency of light whose photons correspond exactly to the work function is known as the cut-off
frequency.
hc /   W  E0 k

 (6, 63  10 )(3  10 ) /   3, 65  10  2, 32  10
34 8 19 19
2)
   3, 50  10
7
m
Ek  E  W0  hc /   W0  (6, 63  10 )(3  10 ) / (3,5  10 )  3, 65  10  2, 03  10
34 8 7 19 19
3) J
4) a) Increasing the intensity merely increases the number of photons in the incident light, and thus the
number of photo-electrons emitted per unit time. It does not increase the frequency of the photons, and
thus there is no change in the energy of a photon.
b) the ammeter reading increases
c) the number of electrons per unit time increases with intensity, and because I = q/t, the current will also
increase.

pg 29
Question 8
1) the photo-electric effect
2) The minimum frequency of light needed to emit electrons from a metal surface.
19 10
n  Q / qe  Q  n  qe  1, 01  10  1, 6  10  1, 616  10
9
C
3)
10 10
Q  I t  I  Q / t  1, 616  10 / 1  1, 62  10 A
4) Decreases – when the intensity of the light is decreased, the number of photoelectrons emitted will
decrease and thus the current strength decreases.
5) a) b) As long as the frequency of the incident light is still greater than the threshold frequency,
photoelectrons will be emitted, but they will have less kinetic energy.
6) The wavelengths of light that are absorbed in the absorption spectrum correspond exactly to the
wavelength of light that is emitted in the line emission spectrum of the same gas.

pg 30
Grade 12
Exam Preparation

Matter &
Materials II
Memorandum

TouchTutor® Series
Physical Sciences

Section A (Multiple Choice Questions)


1) B – know you functional groups well
2) B
3) D – CnH2n+2
4) C – propene has the lowest melting point (-47,6°C), slightly lower than propane (-42°C), because it is a
smaller molecule, with fewer hydrogens. Propanol has a relatively high boiling point at 97°C and propanoic
acid the highest of the four (141°C)
5) C – the double bond in ethene can be broken to accommodate Br in an addition reaction. Butene (with a
double bond) is not saturated, it can have 4 branched isomers. Hydrogenation breaks double / triple
bonds to add hydrogen rather than adding triple bonds.
6) B – there are 5 carbons in the longest chain, a Cl added to the second C in the chain, with one methyl
rd
substituent on the 3 C in the chain.
7) C – 4,4-dibromo-1-butyne or 4,4-dibromobut-1-yne
8) B – ketones, identified by the carboxyl group C=O, or R-C=O-R’
9) C – butanoic acid – functional isomers. The condensed formula for butanoic acid is CH ₃CH₂CH₂-COOH
and for propyl methanoate is C4H8O2. Ethyl methanoate has the formula C3H6O2, methyl ethanoate
(CH3COOCH3), and propanoic acid (C3H6O2).
10) D – while carbon atoms do combine with oxygen, this does not explain the millions of compounds in
organic chemistry. It is rather the carbon bond with other carbons to form increasingly complex molecules.

11) B – tetrahedron
12) B – a homologous series
13) C – a carbonyl group (as shown in diagram)
14) C – pentane has the lowest boiling point, then butan-1-ol (since it has one area available for
stronger hydrogen bonding). Propanoic acid, with two areas for hydrogen bonding) will have the highest
boiling point. This is confirmed in practice: pentane (36,1 °C), butanol (117,4°C) and propanoic acid
(141,2°C)
15) C – a carbon-carbon double bond that can be broken to allow other monomers to bond with it
16) C – addition polymerisation is precisely the addition of monomers in which the C-C double bond
(unsaturated) is broken.
17) B
18) B – 1,1-dichloroethene
19) C
20) D – hex-3-ene

pg 31
Section B (Structured Questions)
Question 1
1) a) carboxylic acids – the functional group identifying this compound as a carboxylic acid is …
Note: the hydroxyl (OH) and carbonyl (C=O) groups together form the carboxyl functional group.
b) The CH=O (or formyl group) ending identifies this as an aldehyde, specifically butanal
c) F is the repeating unit of the polymer polyethene (or polythene or polyethylene) is
ethene. The monomer ethene has the structural formula as shown
2) This is a tertiary alcohol since the OH group is bonded to a tertiary carbon (a carbon
bonded to three other carbons).
3) B and C have the same molecular formula (C4H8O) but a different functional group. In the case of B, the
functional group is the carbonyl group (a ketone), while C has formyl as functional group (an aldehyde)

Question 2
1) a) C is unsaturated, since it contains a carbon-carbon double bond (C=C)
b) Compound A has the molecular formula C4H8O2; methyl propanoate is the same – thus compounds A
and F are structural isomers.
c) Compound D - an aldehyde is identified by the –al ending
d) same homologous series means the same functional group, though different chain length. Compound
B is an alcohol, and so is E (ethanol).

2) a) Compound C is a cycloalkene, with 6 carbons, a double bond on between C 1 and C2, and a methyl
substituent on C3, thus 3-methylcyclohexene
b) Compound B is an alcohol, with the OH (hydroxyl) functional group.
c) Methyl propanoate is an ester.
3) a) Esterification
b) ethyl butanoate
c) Compound A: butanoic acid

Question 3
1) a) A functional group is a bond, atom or group of atoms which identifies to which homologous series a
molecule belongs and is responsible for the chemical properties of that compound.
b) An alkyl substituent is a carbon based side chain which is attached to
the longest continuous chain in the molecule (an alkane group missing one
hydrogen atom, with general formula CnH2n+1)
2) a) CH3-CBr=CH-CHBr-CH2-CH3 — as structural formula:

b) CH3CH(CH3)CH(CH2CH3)CH2CH3 — as structural formula

Question 4
1) a) E
b) A
c) A
d) F
e) A or D
f) C

2) a)
b)

pg 32
3) a) carbonyl group
b) 2-methylpropan-1-ol

Question 5
1) there are two significant elements in the structure: the ring structure and the halogen .. this molecule
belongs to the halocylcoalkenes (alternatives: cycloalkenes, haloalkenes)
2) 1-bromo-2-ethylcyclohex-1-ene (or 1-bromo-2-ethylcyclohexene)
3) the C(=O)H – formyl – group identifies this as an aldehyde, with the ending –al … 2-ethyl-3-methylbutanal
4) propan-1-ol
5) see diagram on right
6) see below

7) B, D, E, G
8) In compound C there is an OH group, and because of the large difference in electronegativity, the H of
the OH group is available for hydrogen bonding. Compound F only has much weaker London dispersion
forces, hence the lower boiling point.
9) Compound B is more branched than compound E, and so has a smaller surface area for London forces to
operate. This means that B has weaker intermolecular forces, meaning that it requires less energy to
break the bonds between the molecules. Hence a lower boiling point.

Question 6
1) a) B (with ending CHO) is an aldehyde
b) E is a condensaton polymer with two different functional groups.
c) F – pentanone (this is a ketone), centred around a carbonyl group connected to two radicals
2) a) 2-bromo-3-chloro-4-methylpentane
b) ethene (remember that in addition polymerisation, the double bond is broken and so there are orbitals to
which the other repeating units can bond to form a long chain)
3) see next page

4) a) functional isomers have the same molecular structure but a different functional group (they belong to
different homologous series)
b) B and F are functional isomers – they have the molecular formula C5H10O, but the one is an aldehyde,
the other a ketone.

Question 7
1) a) carboxyl group (COOH)
b) ketones
c) addition polymerisation
2) a) ethene

pg 33
b) 4-methylhex-3-one
c) 4-ethyl-2,2-dimethylhexane
3) When a hydrocarbon combusts (burns in oxygen), the products are always carbon dioxide (CO 2) and
water (H2O).
4) a) b)

5) a) E – butene is unsaturated and therefore can react immediately with the bromine.
b) This is halogenation (substitution) reaction
c)

pg 34
Grade 12
Exam Preparation

Matter &
Materials III
Memorandum

TouchTutor® Series
Physical Sciences

Section A (Multiple Choice Questions)


-
1) A – propene; this is an elimination reaction. The OH ion attracts and removes a hydrogen atom from the
carbon next to the one holding the bromine. This rearrangement expels the bromine, giving propene as
product.
2) B – Cracking is the process whereby complex organic molecules are broken down into simpler molecules,
by the breaking of carbon-carbon bonds in the precursors.
3) A – the socalled bromine test is a test for unsaturated hydrocarbons. C 2H6 (ethane) is the only compound
here that will not rapidly decolouriae a solution of bromine water since it is a saturated compound.
4) A – this is an example of an addition reaction, specifically hydrogenation, the combination of hydrogen with
unsaturated hydrocarbons. It is also sometimes referred to as a saturation reaction (for obvious reasons)
5) A – ethene readily undergoes halogenation - reacting with a
halogen resulting in the halogen being added to the
compound
6) C – the major product will be propan-2-ol, a secondary
alcohol. The minor product in the hydration of propene is propan-1-ol, a primary alcohol. Use
Markovnikov’s rule to determine the major / minor product.
7) An ester is formed through the reaction of an alcohol and a carboxylic acid. I. – is pentanoic acid, and II. –
is pentanol .. from these two the ester pentyl penatanoate can form. Correct answer is A.
8) B – Step 1 is the hydration of ethene to form ethanol. Step 2 is the full oxidation of ethanol to form
ethanoic acid. When ethanoic acid is reacted with ethanol, esterification takes place and ethyl ethanoate

is formed.
9) C – substitution reaction. Two hydrogen atoms are substituted by two bromine atoms.
10) B – this is a substitution reaction
11) C – London forces. London forces are temporary induced dipoles which can be generated in otherwise
non-polar molecules.
12) E – London dispersion forces – since the noble gases are monatomic and non-polar, and are arranged
with increasing atomic radius.
13) C – C4H10 – since it is the longest hydrocarbon
14) C – 1 and 4 are false statements.
15) D – none of these
16) A – low melting point
17) C – the compounds listed are alkanes, alcohols and carboxylic acids. Of these, alkanes have the lowest
boiling point, followed by alcohols (with their hydrogen bonding) and then carboxylic acids.
18) A – fractional distillation makes use of the different boiling points of hydrocarbons to isolate them.
19) C – compound A is propene. When propene is reacted with a halogen (like Br 2)(halogenation), the
product 1,2-dibromopropane is formed. When propene reacts with water (is hydrated), an alcohol is is
formed. The major product would be propan-2-ol (CH2CH(OH)CH3) rather than propan-1-ol
(CH3CH2CH2OH).
20) D – CCl4 – since it has a tetrahedral structure

pg 35
Section B (Structured Questions)
Question 1
1) It belongs to the alkenes (C=C double bond)
2) a) This is an addition reaction – HCl is added to the original compound. Note that no element /
compound is eliminated. In general terms: A + B → BC
b) The Cl atom is replaced by an OH group to form propan-2-ol
c) This is an example of an elimination reaction – in general terms: A → B + C
d) This illustrates the use of a catalyst in the reaction
3) a) This is an example of a dehydrohalogenation reaction – a hydrogen atom and a halogen are removed
to from an alkene (double bond). The reaction requires the presence of a strong base like NaOH or KOH.
b) Reaction conditions required: the presence of a strong base (already mentioned in a)), the base must
be dissolved in pure ethanol, and it must be heated strongly.
c)

While the issue of major and minor product does not arise here, be aware of it.

Question 2
1) Hydrocarbons are organic compounds that consists of carbon and hydrogen only (alkanes, etc)
2) a) From A to C, the boiling point increases as the surface area available for London forces increases. The
surface area increases as the molecules become less branched, and straighter.
b) The more branches a molecule has, the smaller its surface area, and hence the less opportunity for
London forces to form. With weaker intermolecular forces, less energy is required to separate the chains
(what happens when the substance boils).
3) The lower the boiling point, the higher the vapour pressure, thus compound A will have the highest vapour
pressure.

Question 3
1) a) butanol has four carbons, with an OH group bonded to the first, or last, carbon. Its structural formula is
CH3CH2CH2CH2OH
b) As a first step, dehydrate butan-1-ol to
form but-1-ene, then hydrate but-1-ene
with excess water to form butan-2-ol as major
product and butan-1-ol as minor product.
Note: both reactions require a strong acid
catalyst such as H2SO4, and heat (± 180°C).

2) a) The product in both the hydration or hydrolysis reaction is an alcohol …


(Addition reaction: alkene plus water gives alcohol – hydration / Substitution reaction: haloalkene plus
water gives alcohol – hydrolysis. Remember: both types of reaction involve water as a reactant -
hydration is adding water onto a molecule, hydrolysis is breaking a molecule into multiple pieces by
reacting with water)
b) CH3CH=CH2 + H2O → CH3CH(OH)CH3
c) CH3CH2CH2Cl + NaOH → CH3CH2CH2OH + HCl

Question 4
1) a) A – haloalkane (or alkylhalide), and B – alkene
b) Elimination reaction (the two fluorine atoms are removed)
c) water (H2O)
d) concentrated H2SO4, as catalyst, and pentanoic acid to
react with the ethanol.
e) to belong to a different homologous series, the isomer
needs a different functional group, for example, a carboxyl
group. Heptanoic acid would be a isomer, though there are
many other possibilities.
f) halogenation (or bromination)
g) The bromine liquid loses its colour as the reaction takes place.
pg 36
Question 5
1) a) Since the boiling point (0°C) is below 25°C, compound A will be a gas.
b) Isomers of straight-chain butane (4 carbons) will have more branches and thus be more compact with a
smaller surface area. This implies weaker intermolecular forces and thus a lower boiling point.
c) 2C4H10 + 13O2 → 8CO2 + 10H2O
d) Butane is a non-polar molecule, with only London forces holding the butane molecules together.
Propanal however has a carbonyl group which is polar, and thus greater intermolecular forces. As a
result, propanal will have a higher boiling point than butane even though it has fewer carbon atoms.
2) a)

Ethanoic (acetic acid) has two sites for hydrogen bonding (-OH, =O), whereas propan-1-ol has only
one. As a result, ethanoic acid has a higher boiling point than propan-1-ol.
b) Compound C has a higher vapour pressure. The higher the vapour pressure, the lower the boiling
point.

Question 6
1) Primary alcohol, since the –OH group is attached to a primary carbon (which is attached to only one other
carbon)
2) a) This is an elimination reaction – the
dehydration of an alcohol.
b)
3) a) when an alcohol reacts with a
carboxylic acid, an ester is formed in a
reaction known as esterification.
b)

4) a) this is a substitution reation, with the –OH replaced by Br, a halogen


b) the product is 1-bromobutane.

Question 7
1) I – addition reaction / hydrohalogenation reaction

II – substitution reaction / hydrolysis


III – esterification / elimination reaction
IV – elimination / dehydration reaction
2) a) this is an esterification reaction, butanol reacting with
ethanoic acid to form butyl ethanoate.
b) water (always a byproduct of the esterification reaction)
c) sulfuric acid is used as a catalyst, and to act as a
dehydrating agent (it removes H2O)
d) a warm bath is used since – the butanol is flammable / it spreads the heat more evenly / prevents the
alcohol from vapourising / the butanol does not come into direct contact with a flame.
3) a) C4H8 belongs to the alkenes (CnH2n)
b) CH2=CH-CH2-CH3 (but-1-ene) / CH3-CH=CH-CH3 (but-2-ene) / CH2=C(CH3)-CH3 (methyl propene)

pg 37
4) a) C4H8 + 6O2 → 4CO2 + 4H2O
b) 14 g of C4H8: n = m/M = 14/(12×4 + 8×1) = 0,25 mol of C4H8. This produces 1 mol of CO2.
n = m/M therefore m = nM = 1 × (12 + 2×16) = 44 g. 44 g of CO2 are produced.

Question 8
1) But-1-ene is an unsaturated compound, given the double bond between two carbon atoms.
2) a) b) addition reaction

3) a) 2-chlorobutane
b)

4) a) dilute NaOH (or KOH) and some heat


b) hydrolysis reaction / substitution reaction
c) C4H9Cl + NaOH → C4H10O + NaCl

Question 9
1) a) cracking
b) addition polymerisation
c) far right diagram
2) a)

b) hydrohalogenation
or hydrobromation
note: when the double bond is broken, the bromine attaches to the carbon attached to the greater
number of other carbons (the more substituted carbon), while the hydrogen attaches to the other
carbon.
c)

d) 2-methylbutan-2-ol
e) substitution / hydrolysis
f) mild heat and a dilute solution of a strong base (KOH or NaOH)

pg 38
Grade 12
Exam Preparation

Chemical
Change I
Memorandum

TouchTutor® Series
Physical Sciences

Section A (Multiple Choice Questions)


1) A – a chemical reaction has taken place. B to D all represent physical change
2) A – n(Cu) = m/M = 0,98 / 63,5 = 0,0154 mol Cu. According to the reaction, 2 mol of Al produce 3 mol of
Cu, therefore 0,0154 mol of Cu are produced by 0,01 mol of Al. The rate of Al consumption is 0,01 mol
-3 -1
per 2,5 minutes, or 4,11 × 10 mol·min
3) C – raising the temperature of the solvent will increase the solubility of KCl in water. Stirring the mixture
will only increase the rate of dissolution, as will B, increasing the surface area of the solute. D will have no
effect.
2+ 3-
4) A – Ca forms a Ca ion, and PO4 has a negative 3 charge. To balance charges, three Ca ions are
3-
necessary and two PO4 ions ― Ca3(PO4)2
5) C – in 100g of sample: 63% Mn and 38% O. n(Mn) = m/M = 63/55 = 1,15; n(O) = 37/16 = 2,31. The
ratio of Mn:O is 1,15 : 2,31 = 1:2, therefore the empirical formula is MnO 2.
-1
6) C – aspirin has a molecular mass of 180,157 g·mol . In one 500 mg tablet there would be n = m/M =
-3 -3 23
500×10 / 180,157 = 2,775 ×10 mol. Each mol has 6,022 × 10 molecules, therefore the 500 mg tablet
21
consists of 1,67 × 10 molecules
7) B – 6: the balanced equation is Sn3(PO4)4 + 6H2SO4 → 3Sn(SO4)2 + 4H3PO4
8) B – the definition of a mole: a unit of measurement used in chemistry to express amounts of a chemical
substance, defined as the amount of any substance that contains as many elementary entities (e.g.,
atoms, molecules, ions, electrons) as there are atoms in 12 grams of pure carbon-12. This number is
23
referred to as Avogadro’s number, = 6,022 × 10

9) B – 1, 11, 7, 8: The balanced equation is C7H16 + 11O2 → 7CO2 + 8H2O


10) B – 1019 g: The balanced equation: Ca(OH)2 + Na2CO3 = CaCO3 + 2 NaOH. 1,350 kg of Na2CO3 is
12,74 mol, which, according to the reaction, will produce 25,47 mol of NaOH, with a mass of 1019 g.
11) B – the balanced reaction: (NH4)2S(aq) + Cu(NO3)2(aq) → CuS(s) + 2NH4NO3(aq). n(CuS) = 0,3491 /
-3
95,5 = 3,655 x 10 mol. Based on the balanced reaction, the same number of mol of Cu(NO3)2 are
-3 -3
needed, which in 150,0 mL gives a concentration of c = n/V = 3,566 × 10 / 150 × 10 = 0,0243 or 2,43 ×
-2
10 M of Cu(NO3)2
-1 -3
12) D – glucose (C6H12O6) has a molar mass of 180,16 g·mol , therefore 5,45 x 10 mol of glucose have a
mass of 0,981 g.
2+ -
13) A – iron(II) implies Fe , nitrate has the formula NO3 . To balance the charges, two nitrate ions are
necessary, so iron(II) nitrate has a formula Fe(NO3)2.
14) A – 77% 150 g of O2 = 4,688 mol, 150 g of NH3 = 8,808 mol. Based on the balanced equation, NH3 is
in excess, with oxygen the limiting reactant. 4,688 mol of O 2 theoretically produces 3,75 mol of NO (4,688
× 4/5) which has a mass of 112,5 g. Since only 87 g are produced, the percent yield is 87/112,5 × 100 =
77,3%
15) E – a molecule
16) E – elements
2+ -
17) B – ion. Ions can have a positive or negative charge, e.g. Fe and NO3 . Protons are always positive,
electrons always negative. A neutron has no charge, and an isotope (of a particular element) refers to the
number of neutrons in the nucleus).

pg 39
18) A – hydrogen bond

Section B (Structured Questions)


Question 1
1) a) C3H8 + 5O2 → 3CO2 + 4H2O
b) P4O10 + 6H2O → 4H3PO4
c) S + 6HNO3 → H2SO4 + 6NO2 + 2H2O
d) PCl5 + 4H2O → 5HCl + H3PO4
e) Pb(NO3)4(aq) + 2Na2SO4(aq) → Pb(SO4)2(s) + 4NaNO3(aq)
2) a) Cl: 0 (since it is an element)
b) O: – 1 (Na is more reactive, has an oxidation number of +1, then oxygen must be -1)
c) P: + 5 (F = -1, Na +1, therefore P = 0 – (-6)- (+1) =+5)
d) H: -1
e) N: +5 (O = -2, therefore N2 = 0 – (5 × -2) = + 10, therefore N = +5)
f) Fe: +8/3
g) Fe: +3
h) N: +3
i) N: +4

3) a) hydrofluoric acid
b) nitrous acid (Note: nitric acid is HNO3)
c) carbonic acid (or dihydrogen carbonate)
d) copper(I) phosphate
e) carbon dioxide
f) chromium(VI) carbonate
g) chlorine dioxide
h) sulfurous acid
i) calcium hydroxide

Question 2
+ -3
1) HCl dissociates completely, thus c(H ) = c(HCl) = m / MV = 25 /((1+35,5) × 1,5) = 0,4566 mol·dm .
+
pH = –log[H ] = 0,34
2) Both nitric and hydrochloric acid are strong acids that dissociate completely, therefore
+ -3 +
[H3O ] = (1,2 + 1,7)/1000 = o,oo29 mol·dm . Then pH = – log[H3O ] = 2,538
+ -
3) The solution is neutral – it has a pH of 7. The H ions are balanced by the OH ions present in the
+ - -14 + -7 - -7
solution, since Kw = [H3O ][OH ] = 1 × 10 . If [H3O ] = 1 × 10 , then [OH ] is also 1 × 10 .
4) First calculate the number of mol of KCl present in the 500 mL …

-3
n(KCl) = cV = 2,4 × 500×10 = 1,2 mol.
Now dilute the solution as necessary:
3 3 -3
c = n/V, therefore V = n/c = 1,2 / 1 = 1,2 dm . The total volume must be 1,2 dm , thus 1,2 – 500×10 = 0,7
3
dm of water would need to be added.
5) a) N2(g) + 3H2(g) → 2NH3(g)
b) 25 g of nitrogen: n = m/M = 25/(2×14) = 0,893 mol of nitrogen gas.
1 mol of N2 produces 2 mol of NH3, thus 2 × 0,893 = 1,786 mol of NH3 is produced.
3 3
Each mol of gas at STP occupies 22,4 dm , thus the NH3 has a volume of 1,786 × 22,4 = 40,01 dm .
c) 50,0 L is equivalent to 50,0 / 22,4 = 2,232 mol of NH3. This is produced by 2,232 × 3/2 = 3,348 mol of
H2, which has a mass of 3,348 × 1 = 3,348 g.
6) a) H3PO4 + 3KOH → K3PO4 + 3H2O
-3
b) n(KOH) = cV = 76,4×10 × 0,35 = 0,02674 mol.
-3 -3 3
n(H3PO4) required = 0,02674 / 3 = 8,913×10 mol … in 25,0×10 dm , therefore concentration of
-3
H3PO4 = c(H3PO4) = n/V = 0,357 mol·dm .

Question 3
1) CuCl2 + 2NaNO3 → Cu(NO3)2 + 2NaCl
2) 15 g of CuCl2 = 15/(63,5+71) = 0,112 mol of CuCl2; 20 g of NaNO3 = 20 / (23+14+48) = 0,235 mol
pg 40
Based on the balanced equation, the NaNO3 is in excess (only 0,224 mol are required). CuCl2 is the
limiting reagent, and determines how much product is formed.
Based on equation, 0,112 mol CuCl2 will produce 0,112 × 2 mol of NaCl = 13,104 g of NaCl is produced.
3) CuCl2 is the limiting reagent.
4) 0,235 – 0,224 = 0,011 mol or 0,935 g of NaNO3 will be left over.
5) Percent yield = actual yield / theoretical yield × 100 = 11,3 / 13,104 × 100 = 86,2 %.

Question 4
1) the mole: the mole is defined as the amount of any chemical substance that contains as many elementary
entities, e.g., atoms, molecules, ions, or electrons, as there are atoms in 12 grams of pure carbon-12 (12C),
the isotope of carbon with relative atomic mass 12 by definition.
molar volume: the molar volume, symbol Vm, is the volume occupied by one mole of a substance
(chemical element or chemical compound) at a given temperature and pressure. The molar volume of a
3
gas at STP is 22,4 dm .
-1 -1
2) Molar mass of CuFeS2 is 183,5 g·mol , molar mass of Cu2S is 159 g·mol .
a) 42,3 g of CuFeS2 = 42,3 / 183,5 = 0,2305 mol … this requires twice as many moles of O2, i.e
0,461 mol O2.
b) According to the balanced equation, 2 mol of CuFeS 2 produce 1 mol of Cu2S, thus 42,3 g of CuFeS2
will produce 0,2305/2 = 0,1153 mol Cu2S, with a mass of 0,1152 × 159 = 18,32 g.

c) 2 mol of CuFeS2 produce 3 mol of SO2, thus 42,3 g of CuFeS2 will produce 0,2305 × 3/2 = 0,3458
3
mol of SO2, with a volume, at STP, of 7,745 dm .
d) Based on reactions 1 and 2, 2 mol of CuFeS2 will produce 2 mol of pure Cu, thus
0,2305 mol will produce 0,2305 mol of Cu with a mass of 0,2305 × 63,5 = 14,64 g.
e) high levels of unemployment lead to theft / copper fetches a good price at the scrap yard / copper
has many uses /
-1
3) Molar mass of Li2SO4 = (7×2 + 32 + 4×16) = 110 g·mol .
-3 -3
a) c = m/MV = 7,1 / (110 × 500×10 ) = 0,129 mol·dm
+ 2-
b) Li2SO4 → 2Li + SO4
+ -3
c) [Li ] = 2 × 0,129 = 0,258 or 0,26 mol·dm

Question 5
First consider reaction 2 and determine the volume of H2SO4 that was used (what was in excess from
reaction 1)
na caVa 1 (1)  Va
    Va  7 cm
3

nb cbVb 2 (0, 5)(28)


3
Thus 50 – 7 = 43 cm of H2SO4 was used in reaction 1.
Now calculate the number of moles of H2SO4 used in reaction 1, and from the mole ratio, determine the
moles and mass of the CaCO3 used in the reaction.

-3
n(H2SO4) = cV = (1)(43×10 ) = 0,043 mol.
The ratio of acid to CaCO3 is 1:1, thus n(CaCO3) = 0,043 mol.
m(CaCO3) = nM = 0,043 × (40 + 12 + 3×16) = 4,3 g.

pg 41
pg 42
Grade 12
Exam Preparation

Chemical
Change II
Memorandum

TouchTutor® Series
Physical Sciences

Section A (Multiple Choice Questions)


1) D – a catalyst lowers the activation energy for a reaction (both the forward and reverse reactions)
2) A – a reaction takes place when the collisions between molecules / atoms are effective.
3) A
4) D - draw a vertical line through the letter S … the area of graph S to the right of the vertical line is greater
than that of any other graph P, Q or R. S represents the graph with particles at the highest average kinetic
energy and thus the highest temperature.
5) B – in this reaction, 4 moles of reactants produce 2 moles of product, decreasing the pressure in the
container. The greater the pressure, the more the forward reaction (which reduces pressure) will be
favoured.
6) B – the nitric acid has a higher concentration and so could make the reaction in the second test tube
faster. The volume of the acid plays a very minor role as long as all the zinc is immersed in it. The zinc
pellets have greater mass but less surface area than the powder (so an inhibiting factor). D is false.
7) C – the rate of the forward and of the reverse reaction increases – this happens when a catalyst is added.
8) C – the temperature has been lowered. The graph shows the rate of the reverse reaction dropping
significantly more than the rate of the forward reaction – heat (temperature) has been removed from an
exothermic reaction. The forward reaction is favoured and thus at a faster rate than the reverse reaction,
until equilibrium is reestablished at a lower rate than from 10-15 min.
9) D – equilibrium means that the outward visible signs don’t change, but at microscopic level, both
reactions may proceed. Equilibrium is a dynamic system.

10) C – the concentration of N2(g) has been increased (note, only the concentration of N 2 is increased at t1).
As a result, the forward reaction is favoured (the graph for NH 3 increases), with a consequent decrease in
the concentration of the reactants.
11) D – at equilibrium the rate of the forward reaction is equal to the rate of the reverse reaction.
12) C – high temperature (because it is an endothermic reaction) and low pressure (1 mole → 2 mole).
13) D – a dynamic equilibrium means that at microscopic level, the reactions are still proceeding, though at the
same rate.
14) B – when a solution of CaCl2 is saturated, an equilibrium is reached. If more CaCl2 is added, it will
precipitate out, while if some CaCl2 is removed from the solution, some of the precipitate will dissolve to
restore equilibrium.
15) A – important: equilibrium is not restored, but a new equilibrium is established that partially compensates
for the imposed change.

Section B (Structured Questions)


Question 1
1) HCl is the limiting reagent - there was not enough HCl to react with all the Zn.
2) The flask and its contents lose mass because the hydrogen gas that forms as
a result of the reaction escapes from the flask. To preserve mass, the flask
would need to be sealed with some form of stopper.
3) and 4) due to the higher temperature, the reaction takes place faster, thus
there is a steeper gradient for graph Q than for graph P.

pg 43
5) At a higher temperature, more molecules have kinetic energy higher than the activation energy enabling
more effective collisions to take place per unit time.
6) First calculate the number of moles of HCl present …
-3
c(HCl) = n/V, therefore 0,2 = n / (100 × 10 ), therefore n = 0,02 mol.
According to the balanced reaction, each mol of Zn requires 2 mol of HCl to react … that is, 0,01 mol of
Zn are needed.
The mass of 0,01 mol of Zn: n = m / M, therefore 0,01 = m / (65), therefore m(Zn) = 0,65 g.
This is the mass of Zn that reacted. Add to this the 0,12 g left over after the reaction – thus the amount of
Zn originally in the flask = 0,65 + 0,12 = 0,77 g.

Question 2
1) Equilibrium is reached in a reversible chemical equation if the rate of the forward reaction is equal to the
rate of the reverse reaction.
2) a) at t1, the rate of the forward reaction is higher than the reverse reaction.
b) at t2, the rate of both reactions is equal.
Note: The forward reaction starts at a high rate and then gradually decreases as the concentration of the
reactants decreases. The reverse reaction starts at zero (since there are no products yet), and gradually
increases until the two reaction rates are equal (the point at which equilibrium is reached)
3) a) at t3, the [NO2(g)] increases drastically … NO2(g) is the product of the reaction, and the reaction itself
cannot produce such a drastic increase. The only possibility is that NO 2(g) was added to the container.

b) at t4, the concentration of N2O4(g) decreases while that of NO2(g) increases. The temperature has not
changed (since we are told that the equilibrium constant does not change). The only factor that will disturb
the equilibrium reached just prior to t4 in a forward direction is a decrease in pressure. Note: An increase
in pressure favours the reverse reaction since 2 mol of a gas become 1 mol of another gas.
4) The reaction is endothermic – i.e. it requires energy to proceed in the forward direction. Increasing the
temperature increases the amount of heat available so favouring the forward reaction. Remember too that
increasing the temperature increases the rate of the reactions, therefore the yield per unit time will
increase.
5) Use the RICE method to answer this question …
Reaction N2O4 (g) ⇌ NO2(g)
Mole ratio 1 2
Initial quantity (in mol) 0,92 0
Change (20,7 %) (in mol) 0,92 × 0,207 = 0,19 0,19 × 2 = 0,38
Quantity at equilibrium (mol) 0,92 – 0,19 = 0,73 0,38
-3
Equilibrium concentration (in mol·dm ) 0,73 / 2 = 0,365 0,19
3
(remember: volume = 2 dm )

 NO   0,19 2 2
2
Kc  
2
 0, 0989 or 9, 9  10
 N O  (0, 365)
2 4

Question 3
1) The forward reaction is exothermic. ΔH for the reaction = energy of the products – energy of the reactants
-1
= – 197 – 0 = – 197 kJ·mol . Any value less than zero implies the reaction releases heat, i.e. it is
exothermic.
-1
2) 1580 kJ·mol
-1
3) The total energy involved in the formation of SO3 molecules is –197 – (+1580) = -1777 kJ·mol . The
-1
negative sign implies that energy is released. To answer the question: 1777 kJ·mol of energy is
released.
-1
4) The activation energy for the reverse reaction is the difference from – 197 to 1340 = 1537 kJ·mol .

Question 4
1) The rate of reaction may be defined as the rate of change of the concentration of the reactant P.
Alternatively, the rate of reaction is the change in the concentration of P per unit time.
2) The rate of change is equal to the gradient of the line at a particular point. At 10 s, the gradient of the line
has a greater magnitude (though negative) than at 30 s, thus decomposition takes place at a higher rate at
10 s. This is perfectly reasonable – when there is more reactant, the reaction rate is greater than when
lots of the reactant has already been decomposed.
-3
3) Each unit on the vertical axis represents 0,005 mol·dm . The y-intercept is four units above 0,25, thus at
-3
0,27 mol·dm .
st -1
4) Ave. rate of decomposition in 1 10 s in mol·s = number of moles of P decomposed per unit time …

pg 44
n(P) at 0 s: n = cV = 0,27 × 2 = 0,54 mol.
n(P) at 10 s: n = cV = 0,15 × 2 = 0,3 mol.

rate of change  0, 3  0, 54  0, 024 mol  s


-1

10  0
5) see graph
6) At a higher temperatures, there will be more reactant molecules with
sufficient energy to allow for more effective collisions per unit time.
Consequently, an increase in temperature will increase the rate of the
decomposition of P.

Question 5
1) Erlenmeyer or conical flask
2) The syringe collects the H2 gas produced, and is also used to measure the amount or volume of H 2
produced.
3) a) the concentration of the hydrochloric acid (HCl) used is the independent variable
b) the temperature, held at a constant 25°C, is a controlled variable. Also controlled is the surface area of
the zinc, since it is used in powdered form in both experiments.
4) The graph P represents experiment 2. In experiment 2, a higher concentration of acid is used. This leads
to a faster initial rate of hydrogen production, represented by the initial steeper gradient of graph P.

5) As the reaction proceeds, the concentration of the acid (in both experiments) decreases, thus the gradient
(the production of H2 gas per unit time) decreases.
3 3
6) At room temperature, one mole of a gas has a volume of 24,04 dm . If 0,24 dm of hydrogen gas are
formed, then 0,24 / 24,04 = 0,00998 or 0,01 mol of hydrogen gas (H 2) was formed.
Based on the balanced equation, 1 mole of H2 gas is formed for each mole of Zn consumed in the
reaction.
The mass of Zn used = nM = 0,01 × 65 = 0,65 g.

Question 6
1) Zn(s) + 2HCl(aq) → ZnCl2(aq) + H2(g)
2) The surface area available for reaction affects the rate of reaction
3) The amount of zinc used must be controlled to make it a fair experiment.
4) The excess hydrochloric acid will in no way affect the amount of hydrogen gas produced since the volume
of hydrogen gas produced is, with the excess of HCl, dependent only on the amount of zinc used.
5) The average rate of reaction = change in volume of hydrogen gas produced per unit time ..
V 48  30
   0, 9 cm  s
3 -1

t 40  20
6) The average rate of reaction will be greater in experiment 2.

7) In experiment 2, zinc is used in powdered form and thus has a much greater surface area open to reaction
than to the zinc granules used in experiment 1. With a greater surface area, more effective collisions can
occur per unit time.
8) a) the volume of hydrogen gas produced remains the same
b) the gradient of the graph will be steeper.
9) The copper added to the reaction acts as a catalyst. The catalyst lowers the activation energy, and thus
increases the rate of reaction.

Question 7
1) This is an example of a homogeneous equilibrium since the reactants and products are in the same phase
and cannot be distinguished from each other.
2) A and C both take place at 500°C. The only factor that can speed up the reaching of equilibrium and
faster reaction rates (for both forward and reverse reactions) is a catalyst
3) The only difference between A and C is that C reaches equilibrium faster. The concentration of products
and reactants in both A and C are the same at the beginning of the reaction and when equilibrium is
reached.
4) The reaction must be exothermic.
5) From A to B, the temperature was increased from 500°C to 600°C. This has shifted the equilibrium so that
the concentration of the products in B is less than the concentration of the products in A. Thus the reverse
reaction was favoured by the increase in temperature (as was the rate of both reactions). According to Le

pg 45
Chatelier’s principle, an endothermic reaction is favoured by an increase in temperature, i.e. an exothermic
reaction is not favoured. The reverse reaction is endothermic, the forward reaction is exothermic.
6) First calculate the number of moles of reactants and products present at the beginning of the reaction in A.
n(H2) = ½ = 0,5 mol; n(I2) = 127/(127 × 2) = 0,5 mol.
Reaction: H2(g) + I2(g) ⇌ 2HI(g)

H2(g) I2(g) HI(g)


Mole ratio 1 1 2
Initial number of mole present 0,5 0,5 0
Change (note: concentration of HI –
3 –0,4 –0,4 +0,4 × 2
product in 2 dm )
Number of mole at equilibrium 0,5 – 0,4 = 0,1 0,5 – 0,4 = 0,1 0,8
-3
Equilibrium concentration (mol·dm ) 0,1 / 2 = 0,05 0,1/2 = 0,05 0,8/2 = 0,4

 HI 2
(0, 4)
2

Kc    64
 H  I  (0, 05)(0, 05)
2 2

Note: the large value of Kc implies the reaction equilibrium lies significantly to the right .. a large amount of
product is formed.

Question 8
1) Reaction rate may be defined as the rate of change of the concentration of the products (or of the
reactants). Note, rate of change, i.e. change per unit time. Instead of concentration, one may use the
amount (number of moles) of product / reactants, etc. There is no one definition of reaction rate.
2) a) The temperature is changed, thus it is the independent variable
b) Dependent variables: the volume of gas formed per unit time.
3) With a larger mass of powdered CaCO3, there is an increase in the surface area where effective collision
may take place. The larger the number of effective collisions per unit time, the higher the rate of reaction.
4) Compare the various graphs with the data for experiments 1, 2, 3 and 4.
In experiment 1, a higher concentration of acid is used than in experiment 2. All other factors remain
constant. A higher concentration would imply a higher rate of reaction and thus a steeper initial gradient
for the production of CO2. Thus graph D (with the lowest gradient) cannot represent experiment 1.
In experiment 3, a higher temperature is used than in experiment 1. A higher temperature will lead to a
higher rate of reaction, thus a steeper gradient. Similarly, experiment 4 has a higher rate of reaction due
to the larger amount of impure powdered CaCO3, thus a steeper initial gradient. This implies that the
graph representing experiment 1 cannot be A or B.
Considering the above, the graph representing experiment 1 is C.
5) First determine the number of moles of CO2 that are formed …
n(CO2) = V/Vm = 4,5 / 25,7 = 0,175 mol.
Based on the balanced equation, 1 mol of CaCO3 produces 1 mol of CO2, thus we need 0,175 mol of

CaCO3.
The mass of 0,175 mol of CaCO3: m = nM = 0,175 × 100 = 17,5 g.
The initial mass of impure CaCO3 was 25 g, thus 25 – 17,5 = 7,5 g did not react – the mass of the
impurities in the CaCO3.

Question 9
1) Equilibrium is reached in a reversible chemical equation if the rate of the forward reaction is equal to the
rate of the reverse reaction.
N O  N O 
Kc  171    N 2 O 4   6, 84 mol  dm
2 4 2 4 -3
2)
 NO  2
2
(0, 2)
2

3 -3
At equilibrium, the number of moles of N2O4 in 80 cm = cV = 6,84 × 80×10 = o,5472 mol. This means
that the number of moles formed (the change) of N2O4 is 0,547 moles.
3 -3
The number of moles of NO2 at equilibrium (also in 80 cm ) = cV = 0,2 × 80×10 = o,016 mol. Based on
the balanced equation, each mol of N2O4 requires 2 mol of NO2, thus there was a change of 0,547 × 2 =
1,094 moles of NO2. If there were 0,016 mol at equilibrium, there then must have been 0,016 + 1,094 =
1,11 mol of NO2 at the beginning.

You can also use the standard RICE method, working backwards …

pg 46
NO2 N2O4
Mole ratio 2 1
-3
Equilibrium concentration (in mol·dm ) 0,2 6,84
3
Quantity at equil. (mol)(in 80 cm : × 0,08) 0,2 × 0,08 = 0,016 6,84 × 0,08 = 0,5472
Change (mol) 0,547 × 2 = 1,094 0,547
Initial quantity (in mol) 1,094 + 0,016 = 1,11 0
3) a) The reaction mix will appear darker because the concentration has been increased and the molecules
are closer together.
b) An increase in pressure favours the forward reaction (which would relieve the pressure, there being
fewer molecules). The number of moles of NO2 decreases, and thus the dark brown colour of the initial
reaction mix will lighten (N2O4 being a colourless gas)
4) It is an exothermic reaction. An increase in temperature favours the reverse reaction.
a) The colour of the reaction mixture will become darker, for the simple reason that an increase in
temperature favours the reverse reaction.
b) A change in temperature is the only thing that will change an equilibrium constant. Being an exothermic
reaction, an increase in temperature will lower the equilibrium constant (less product will be formed at
equilibrium)

Question 10
1) A reversible reaction is a chemical reaction where the reactants form products that, in turn, react together
to give the reactants back. Reversible reactions will reach an equilibrium point where the concentrations
of the reactants and products will no longer change.
2) no change – since equilibrium has been reached. (Note: the concentrations don’t change, but this does
not mean that the reactions are no continuing. They are continuing, but at the same rate)
3) a) the temperature of the system has been decreased.
b) a decrease in temperature will lower the rate of both the forward and the reverse reactions. However, a
decrease in temperature will also favour an exothermic reaction (the reverse reaction in this case), which
will thus experience less of an impact on its reaction rate than for the forward reaction. As the graph
shows, the forward reaction rate drops suddenly and significantly by comparison to the reverse reaction.
 H  I 
4) Kc  2
 0, 02
2

 HI 2

At equilibrium, [HI] = 0,04/2 = 0,02 mol·dm-3


and [H2] = [I2]. Let [H2] = x, then …
2
x 6 3
0, 02   x  8  10  x  2, 83  10 mol  dm
2 -3
2
(0, 02)
-3 -3
The concentration of H2 = 2,83 × 10 mol·dm

5) The equilibrium constant for the reverse reaction = 1/Kc for forward reaction …
Kc = 1 / 0,02 = 50
6) The forward reaction is endothermic, therefore the increase in temperature will increase the equilibrium
constant for the forward reaction.

Question 11
1) a) if a single lump of the same mass of MgCO3 is used, the reaction rate will decrease and the rate of gas
production will be decreased.
b) Increasing the temperature will increase the rate of reaction, and thus increase the rate of production of
gas.
c) the HCl is in excess (the magnesium powder is fully immersed in the acid solution), and adding extra
HCl as the same molarity will not change anything.
2) Using a single lump rather than a powder will decrease the surface area of the reactant, thus decreasing
the possibility of effective collisions.
3) a) the shaded area represents the number of particles with energy greater than the activation energy, thus
particles that can readily take part in the reaction.
b) firstly, particles must collide (no collision, no reaction) / secondly, the particles colliding must do so with
sufficient energy to react / thirdly, the particles must have the correct orientation.

pg 47
c) a catalyst lowers the activation energy so that more particles
have sufficient kinetic energy to participate in the reaction.
d) Increasing the concentration of the hydrochloric acid increases
the number of particles per unit volume. This increases the
possibility of collision, thus leading to more effective collisions and
thus a faster reaction rate.

Question 12
1) Activation energy
2) a) Graph C represents the higher temperature. For graph C, the area X is much larger than for graph A –
this implies that for C, there are more molecules with high kinetic energy than for A. Temperature is a
measure of the average kinetic energy of the molecules / particles of a substance, thus C, with more
higher energy molecules) represents a higher temperature.
b) An increase in temperature increases the average kinetic energy of the molecules. This means that
more molecules have sufficient energy to effect effective collisions, i.e. there are more effective collisions
per unit time. Thus the rate of reaction increases.
3) a) a positive catalyst is any substance that speeds up a chemical reaction (or allows it to proceed) without
itself undergoing permanent change.
b) and c) A catalyst would lower the activation energy (creating an alternative reaction path), moving the
line Y to the left. It would enlarge the area X.

pg 48
Grade 12
Exam Preparation

Chemical
Change III
Memorandum

TouchTutor® Series
Physical Sciences

Section A (Multiple Choice Questions)


1) D – NaOH (red litmus will turn blue when dipped into a base solution).
2) The end point is point at which the indicator changes colour (at 4,6) while the equivalence point is the point
at which the acid and base are completely neutralised (pH between 8 and 9). Clearly they don’t occur at
the same time. The indicator will be blue at the equivalence point (and long before). The endpoint occurs
before the equivalence point – so correct answer is C.
3) B – CH3COOH can be a proton donor but not a proton acceptor.
4) B – 0,1250 M: the chemical reaction is 2 NaOH + H2SO4 = Na2SO4 + 2 H2O, i.e. for every 2 mol of NaOH,
-3 -3
1 mol of H2SO4 is required. n(NaOH) = cV = 0,5 × 21,17×10 = 0,011 mol, therefore 5,29×10 mol of
-3 -3
H2SO4 are required. [H2SO4] (the concentration of H2SO4) c = n/V = 5,29×10 / 42,35×10 = 0,125 M.
-3 -3
5) D – 0,0085 mol. n(HCl) = cV = 0,35 × 24,35×10 = 8,52 × 10 mol or 0,0085 mol
6) D – the balanced equation for the titration: HCl + NaOH ⇌ NaCl = H2O; this shows that every mol of HCl
requires one mol of NaOH. Thus 0,0085 mol of NaOH will be present at the end point of the titration.
+ - -14 - + + -pH -9
7) A – Kw = [H3O ][OH ] = 1 × 10 , therefore [OH ] = Kw/[H3O ]. If pH = 9, then [H3O ] = 10 = 10 .
- -5
Thus [OH ] = 10 M
8) C – the equivalence point for this titration lies around a pH of 5 which indicates that a strong acid and a
weak base have been used for this titration.
9) C – 5,3
10) D – methyl red, that changes around a pH of 5. Bromothymol blue changes around a pH of 7 - 8, and

phenolpthalein around pH of 8 - 9
+ + -4,3 -5
11) A – pH = -log[H3O ], therefore [H3O ] = 10-pH = 10 = 5,01 × 10 M
12) A – HCl, since a Arrhenius acid is defined as a substance that when added to water increases the
+ +
concentration of H (H3O ) ions present
13) A – the hydrogen ion and hydroxide ion will combine to form water.
14) A – acid should always be added to water, rather than the other way round. This is a safety precaution
applicable particularly to sulfuric acid. The mixture should be stirred throughout to spread the heat of
reaction.
– 2–
15) D - HC2O4 is the acid, losing a proton to become C2O4 (the conjugate base)
16) B – the substances that accepts a proton is the Brønsted-Lowry base. H2O accepts a proton to become
+ 2–
H3O . Similarly, C2O4 is a Brønsted-Lowry base.
17) A – amphiprotic. Diprotic means to be able to give off 2 protons, miscible means that it mixes.
-14
18) B – Kw for water is 1 × 10 , a very small value.
+ -
19) Brønsted-Lowry bases are proton acceptors, which are NH3 (accepting a proton to become NH4 ) and OH
(accepting a proton to become H2O in the reverse reaction). Correct answer is D.
20) D – HCl is the strongest acid, followed by CH3COOH. KOH is the strongest base.

pg 49
Section B (Structured Questions)
Question 1
+
1) A substance is an acid if, dissolved in water, it increases the concentration of the hydronium (H 3O ) ions.
2) Ethanoic acid is a weak acid since it does not dissociate completely (it is only partially ionised) – the
- +
equilibrium for the reaction CH3COOH + H2O CH3COO + H3O lies far to the left.
3) A solution containing a precisely known concentration of an element or a substance
4) Equipment that is needed for accurate measurement: a burette, a pipette. Other equipment needed
(though they don’t influence accuracy) are retort stand, flasks, water bottle, white tile, etc.
-3
5) NaOH is a strong base and dissociates completely. Thus NaOH at a concentration of 0,5 mol·dm
-3 -
produces a concentration of 0,5 mol·dm of OH ions.
+ - -14 + -14 -14
Kw = [H3O ][OH ] = 1 × 10 ; therefore [H3O ] = 1 × 10 / 0,5 = 2 × 10 .
-14
pH = – log[H3O+] = –log(2 × 10 ) = 13,7
Note: NaOH is a strong base, so the pH should be between 12 and 14, so the answer is perfectly
reasonable. Always double-check and confirm the answers to calculations that you do to ensure the
answer is indeed reasonable.
b) First calculate the number of moles of NaOH used to neutralise the vinegar. This will give you the
number of moles of ethanoic acid (mole ratio is 1:1), and then calculate its mass.
-3
n(NaOH) = cV = 0,5 × 40×10 = 0,02 mol, therefore 0,02 mol of CH3COOH were neutralised.
m(CH3COOH) = nM = 0,02 × (12×2 + 16×2 + 1×4) = 1,2 g.

Percentage mass ( % mass) of CH3COOH = 1,2 / 20 × 100 = 6%


6) Hydrolysis is a reaction involving the breaking of a bond in a molecule using water. The reaction mainly
occurs between an ion and water molecules.
- -
CH3COO (aq) + H2O(ℓ) → CH3COOH(aq) + OH (aq)

Question 2
1) a) C – Na2SO4 is a neutral salt that is derived from a strong acid (sulfuric acid) and a strong base (sodium
hydroxide). As such, the solution of sodium sulfate will have a pH of 7. H 2SO4 and HCl are strong acids
with a pH lower than 7, while Na2CO3 and NaOH are bases, with a pH higher than 7. Na2CO3 is a salt of a
weak acid (carbonic acid) and a strong base, NaOH
+ +
b) A – H2SO4 will have the highest concentration of H3O ions – with a pH around 1. HCl (with fewer H
+
ions to donate) will have a lower concentration of H3O .
c) E – NaOH is a strong base
d) B – HCl is the only monoprotic acid in the list.
+
2) We assume that H2SO4 dissociates completely, donating 2 H ions for each molecule of the acid. The
+ 2-
reaction: H2SO4 + 2H2O → 2H3O + SO4
+
pH = –log[H3O ] = –log(2 × 0,1) = 0,699 or 0,7
+ 2-
3) Na2CO3, a salt, dissociates into its component ions Na and CO3 . In water, the reaction for the salt is:
2- - -
CO3 + H2O → HCO3 + OH
-
The presence of the OH ions turns the litmus paper blue (indicating a basic solution, not an acidic one).

Question 3
1) An acid is said to be diluted when it contains a small number of moles of the acid in relation to the volume
of water. To dilute a solution means to add more solvent without the addition of more solute.
-1
2) c = n / V, and since n = m / M, c = m / MV. The molar mass of KOH = (39 + 16 + 1) = 56 g·mol . Thus
m
0, 2  3
 m  3, 36 g
(56)(300  10 )
3) An indicator is a chemical compound that changes the colour of a solution across a certain pH range. The
indicator thus shows when the pH of the solution being titrated changes. The point at which the colour
changes is called the end point.
c AVA nA c A  20,15 1
    c A  0,0746 0,075 mol  dm
-3
4) Using the formula provided: or
cBVB nB 0, 2  15, 04 2
You should be able to calculate the result from first principles …
The reaction: 2KOH(aq) + H2SO4(aq) → K2SO4(aq) + 2H2O(ℓ)
-3
c(KOH) = n/V, therefore n(KOH) = 0,2 × 15,04×10 = 0,003008 mol.
For each mole of KOH, ½ mol of H2SO4 is required, thus 0,001504 mol of H2SO4 are needed.
-3 -3
c(H2SO4) = n/V = 0,001504 / 20,15×10 = 0,075 mol·dm .
5) Bromothymol blue is the most suitable indicator. The titration involves a strong acid and a strong base,

pg 50
which means that the equivalence point (when the acid has been neutralised by the base) is at a pH of
around 7. Since bromothymol blue is the only indicator that changes colour at this point, it is the only
suitable indicator to use.

Question 4
1) a) HNO3 dissociates completely in solution, hence the classification as a strong acid.
b) NO3- (the nitrate ion) is the conjugate base.
c) pH = – log[H3O+] = – log(0,3) = 0,52
-3
2) a) c(HCl) = n/V; therefore n(HCl) = cV = 2 × 100×10 = 0,2 mol.
b) Q is a burette
c) A strong acid (HCl) and a strong base (NaOH) are titrated. The equivalence point will be reached at
around a pH of 7, thus the endpoint of the titration (where the indicator changes colour) should be around
a pH of 7. Based on this, indicator B would be the only suitable indicator.
d) Adding distilled water does not increase the number of moles of the acid in the Erlenmeyer flask and so
will not have any effect on the titration.
-3 -3
e) c = n/V, therefore n(NaOH) = cV = 0,2 × 21×10 = 4,2 × 10 mol.
-3
Since HCl and NaOH react in a ratio of 1:1, the excess HCl consisted of 4,2 × 10 mol.
f) From the previously calculated results, determine the number of moles of HCl that reacted with the MgO.
Initial moles of HCl (0,2) – moles of HCl in excess (0,0042) = 0,1958 mol (= number of moles that reacted

with MgO)
MgO and HCl react in a ratio 1:2, thus 0,1958 / 2 = 0,0979 mol of MgO reacted with the hydrochloric acid.
The mass of MgO: m(MgO) = nM = 0,0979 × (24+16) = 3,916 g.
The percentage purity = mass of MgO reacting / mass of sample × 100 = 3,916 / 4,5 × 100 = 87,02 %.

Question 5
1) A strong acid is one that dissociates completely in solution.
+ + -pH + -3 -3
2) a) pH = – log[H3O ] … the inverse: [H3O ] = 10 . Thus [H3O ] = 10 mol·dm
b) and c) A pH of 3 indicates a moderately strong acid … HCl generally has a lower pH, between 0 and 2.
The HCl acid mentioned has a pH = –log(o,01) = 2, thus the unknown acid is weaker than HCl.
3) a) see above ..
+ -
b) NH4Cl + H2O → NH3 + H3O + Cl
+
c) The presence of the H3O ions indicates that this will be an acidic medium, thus the methyl orange
indicator will be red.
4) First consider reaction 2 and determine the volume of H2SO4 that was used (what was in excess from
reaction 1)
na caVa 1 (1)  Va
    Va  7 cm
3

nb cbVb 2 (0, 5)(28)

3
Thus 50 – 7 = 43 cm of H2SO4 was used in reaction 1.
Now calculate the number of moles of H2SO4 used in reaction 1, and from the mole ratio, determine the
moles and mass of the CaCO3 used in the reaction.
-3
n(H2SO4) = cV = (1)(43×10 ) = 0,043 mol.
The ratio of acid to CaCO3 is 1:1, thus n(CaCO3) = 0,043 mol.
m(CaCO3) = nM = 0,043 × (40 + 12 + 3×16) = 4,3 g.

Question 6
1) a) an acid is a proton donor
+
b) Sulfuric acid (H2SO4) can donate 2 H ions, and thus is diprotic (di meaning two)
2) a) amphiprotic
- -
b) H2CO3 is the conjugate acid for the base HCO3 . HCO3 accepts a proton (it is a base) to form H2CO3.
3) First, a note of caution: never neutralise acid spilled onto a person with a base – you’re likely to do more
damage and miss lots of acid that then is not neutralised. Instead, wash the acid off the lots of water.
a) To calculate the volume of sulfuric acid, first calculate the number of moles of NaHCO 3 (sodium
hydrogen carbonate) that were used.
n(NaHCO3) = m / M = 27 / 84 = 0,321 mol.
n(H2SO4) needed = ½ n(NaHCO3) = ½ × 0,321 = 0,1605 mol.
3
c = n / V, therefore V = n/c = 0,165 / 6 = 0,028 dm
pg 51
3 -3
b) A 1 dm of 0,1 mol∙dm solution of sulfuric acid would have (c = n/V) 0,1 mol of H2SO4.
-3
c = n/V, or V = n/c … thus 0,1/6 = 0,0167 dm3 of the 6 mol∙dm solution of sulfuric acid are
needed.
c) The indicator shows the end point of a titration by a change in colour.
3
d) Calculate the moles of H2SO4 in the 25 cm .
-3 -3
n = cV = o,1 × 25×10 = 2,5 × 10 mol.
+
Since the dissociation of sulfuric acid produces two H ions for every molecule of H2SO4,
-3 +
there are 5 × 10 mol of H ions in the flask before the addition of any sodium hydroxide.
+ -
Some of these H ions are neutralised by the NaOH (or OH ions) that are added ..
3 -3 -3
In 30 cm of NaOH: n(NaOH) = cV = 0,1 × 30×10 = 3,0 × 10 mol NaOH … and since NaOH is a
-3 -
strong base, and dissociates completely, there are 3,0 × 10 mol OH ions.
+ -
H + OH → H2O – the neutralisation reaction
+ + - -3 3
H left over = n(H ) – n(OH ) = 2 × 10 mol, in 25 + 30 = 55 cm .
+ -3 -3 -3
[H ] = n/V = 2 × 10 / 55 × 10 = 0,036 mol·dm
+
therefore pH = – log[H ] = 1,44

Question 7
1) a) (i) the rate of reaction (ii) concentration

b) As the concentration of Na2S2O3(aq) increases, the time decreases.


c) HCl is the limiting reagent since, based on the balanced chemical equation, double the amount of HCl is
needed to react with all the Na2S2O3.
2) a) experiment 6 – since it took place at the highest temperature.
b) if the temperature of the reaction mixture increases, the average kinetic energy of the particles is
higher. Thus more particles will have sufficient kinetic energy to collide effectively. Thus the number of
effective collisions per unit time increases, the rate of reaction will increase.

Question 8
1) The endpoint is the point at which the indicator changes colour.
-1 -3
2) M[KOH] = 56 g·mol . n = m / M = 0,28 / 56 = 5×10 mol KOH.
na: nb = 2:1, thus 0,005 mol KOH react with 0,0025 mol H2SO4.
-3 -3
c = n / V = 0,0025 / 20×10 = 0,125 mol·dm
3) a) it is an acid-base reaction (or neutralisation)
b) HNO3 + NH3 → NH4NO3
c) acidic
+ -
d) NH4NO3 + H2O → NH3 + H3O + NO3
+
the presence of H3O ions makes the solution acidic.

pg 52
Grade 12
Exam Preparation

Chemical
Change IV
Memorandum

TouchTutor® Series
Physical Sciences

Section A (Multiple Choice Questions)


- + 2+ 2+
1) B – E°cell = E°red – E°oxidation = 0,34 – (-0,91) = 1,25 V. 2I | I2 || Ag | Ag: 0,26 V; Mg | Mg || Mn | Mn:
2+ 2+
1,18 V; Pb | Pb || Hg | Hg: 0,98 V
2) A – an electrolytic cell is distinguished from a galvanic cell by the fact that current is applied to achieve
some chemical reaction.
3) Sn is oxidised, and oxidation takes place at the anode, thus A is correct.
+
4) Reduction takes place at the cathode, and H is reduced to H2. The correct answer is D.
2+ - 2+ 2+
5) A – Cd + 2e ⇄ Cd lies above Ni , but below Zn on Standard Reduction Potential Table. Mg lies
above both and Cu and H2 below both.
6) C – Of Cu, Ag, Hg and Mg, only magnesium lies above the hydrogen reaction on the Standard Reduction
Table, indicating that this reaction is spontaneous.
7) C – H2SO4 is used in the reaction, thus the relative density of H2SO4 in the battery decreases. The
concentration of H2SO4 does not increase, nor is it oxidised or reduced.
3+
8) A – aluminium ions (Al ) is reduced to Al rather than oxidised.
2+
9) C – the reducing agent is oxidised. Zn is oxidised to Zn and thus is the reducing agent.
10) B – in a galvanic cell, electrons from from the anode to the cathode via the external wire.
2+ - 2+
11) D – when CuCl2 is dissolved in dissolved in water, Cu and Cl ions are present. Cu is reduced to Cu at
-
the cathode, and Cl is oxidised to form Cl2 at the anode.

12) B – the salt bridge ensures electrical neutrality in a cell through the movement of ions.
13) D – of the elements listed, magnesium is the one lowest of the reduction table, i.e. will more readily be
oxidised and so prevent the iron pipe from rusting.
14) A – Cl has an oxidation number of +3, Cr has an oxidation number of +6, P is at +5 and Mn at +7
15) B – Cl2 gas is pumped over the platinum electrode, gaining electrons – it is reduced (at the cathode). The
+ -
anode half-reaction is the oxidation of silver, Ag → Ag + e
16) C – the silver anode slowly goes into solution, but the platinum electrode does not undergo any change.
2+ 2+
17) C – Ni is lower on the Standard Reduction Table than Cu , therefore it will be oxidised.
18) C – at equilibrium, the concentrations of the reactants and products have become constant and cell
reactions have ceased. There is no excess or deficiency of electrons in either electrode, and the potential
difference between them is zero.
19) A – 0,61 V. E°cell = E°reduction – E°oxidation = 0,34 – (-0,27) = – 0,61 V
2+
20) A – Ni is oxidised to Ni , and oxidation takes place at the anode. Electrons flow from the anode to the
cathode

pg 53
Section B (Structured Questions)
Question 1
+ -3
1) Standard conditions imply that the concentration of the H is 1 mol·dm
2) Iodine does not conduct electricity since each molecule of iodine comprises two iodine atoms joined by a
covalent bond that cannot be excited sufficiently to transfer electrical energy.
3) graphite is a good conductor of electricity, and it is inert, so will not react.
4) a) Here is a list of the substances / elements with their oxidation numbers before & after reaction ..
+
H: H (+1) → H2O (H: +1) – no change in the oxidation number
-
I: I (-1) → I2 (0) – iodine is oxidised
O: MnO4 (-2) → H2O (-2) – there is no change in the oxidation number of oxygen
- 2+
Mn: MnO4 (+8) → Mn (+2) – Mn is reduced.
- -
The oxidising agent is itself reduced, thus MnO4 must be the oxidising agent. The name for MnO4 is
permanganate ion.
- + - 2+
b) 2MnO4 (aq) + 16H (aq) + 10I (aq) → 2Mn (aq) + 5I2(s) + 8H2O(ℓ)
c) About cell notation: the electrodes should be on the outside; the salt bridge is represented by a ||; the
anode (oxidation reaction) is written on the left, the cathode (reduction reaction) on the right. The H+
indicates that the reduction reaction takes place in an acid medium.
- + - 2+
C(s) | I (aq) | I2(s) || H (aq), MnO4 (aq) | Mn (aq) | C(s)

Cell potential: E   Ecathode  Eanode  1, 51  ( 0, 54)  0, 97 V


o o
5)
-
6) As the concentration of the MnO4 decreases, the reading on the voltmeter will decrease.

Question 2
1) An electrolyte is a substance that conducts electricity, i.e. a solution containing ions that can conduct
electricity, or a substance, that when dissolved in water, conducts electricity.
3+
2) the bracelet is the cathode … to plate the bracelet with chromium, the Cr ions must be reduced to Cr on
the bracelet. This is a reduction reaction that takes place at the cathode (remember Red Cat)
3) a) Electrode X is made of chromium, to supply the necessary chromium ions that will be deposited on the
bracelet in the plating process.
3+ -
b) Cr (aq) + 3e → Cr(s)
4) Calculate the # of mol of Cr in 0,86 g, then multiply by 3 (for each Cr atom, 3 electrons are required).
n(Cr) = m / M = 0,86 / 52 = 0,0165 mol.
23 22
number of electrons = 0,0165 × 3 = 0,0496 mol = 0,0496 × 6,02 × 10 = 2,987 × 10 electrons.

Question 3
1) Electrolysis is the chemical decomposition produced by passing an electric current through a liquid or
2+
solution containing ions. In our question, Cu ions are reduced to Cu atoms (by accepting electrons from
-
the negative electrode (B), while Cl ions are oxidised to form Cl2 gas (by giving away their extra electron

at the positive electrode (A). Oxidation takes place at the anode, thus electrode A is the anode here.
2) Since power is required in the form of a battery providing a potential difference across the electrodes, the
process is endothermic.
3) Chlorine gas forms at the anode, electrode A.
Solid copper is deposited on the cathode, electrode B.
The pale blue colour of the copper chloride solution (due to the copper ions) gradually disappears as the
2+
number of Cu ions decreases.
4) CuCℓ2(aq) → Cu(s) + Cl2(g)
5) CuCl2 must be in solution since otherwise the ions are not able to move in order to conduct electricity.

Question 4
1) a) In this electrochemical cell, chemical energy is converted into electrical energy.
b) From the table of standard electrode potentials, Cu has a greater reducing ability than Ag, i.e. Cu will be
2+ +
oxidised to Cu at the anode, while Ag is reduced to Ag at the cathode. Thus the reaction is
+ 2+
Cu(s) + 2Ag (aq) → Cu (aq) + 2Ag(s)
2+ +
c) Cu(s)|Cu (aq)||Ag (aq)|Ag(s)
E   Ecathode  Eanode  0, 80  (0, 34)  0, 46 V
o o
2)
3) from Cu to Ag; Cu atoms from the copper electrode lose two electrons and these move through the
external circuit to the Ag electrode where they are accepted by the Ag+ ions.
pg 54
4) While the copper electrode loses 3,2 g, the silver electrode will gain mass.
Number of Cu atoms lost = n(Cu) = m / M = 3,2 / 63,5 = 0,05 mol.
+
For each mol of Cu atoms oxidised, 2 mol of Ag ions are reduced to Ag, i.e. 0,05 × 2 = o,1 mol Ag.
m(Ag) = nM = 0,1 × 108 = 10,8 g.
Thus the silver electrode gains 10,8 g in mass from silver atoms deposited on it.
+
5) a) H2 has a greater reducing ability than Cu (it is a weaker reducing agent), thus H 2 will be oxidised to H
2+
at the anode, while Cu is reduced to Cu at the cathode. The copper electrode is positive.
b) the emf of the standard hydrogen half-cell is zero, thus the emf of the cell is 0,34 V.

Question 5
1) The pressure of the hydrogen gas bubbled over the platinum electrode must be 1 atmosphere (101,3 kPa),
and the system must operate at a temperature of 25°C.
2) Salt bridge
3) Magnesium is a much stronger reducing agent than hydrogen gas, consequently it will be oxidised.
Oxidation takes place at the anode, thus the magnesium electrode is the anode.
2+ -3 + -3
4) Mg(s)|Mg (1 mol·dm )||H (1 mol·dm )|H2(g)|Pt(s)
E   Ecathode  Eanode  2, 36  0, 00  Eanode  Eanode  0, 00  2, 36  2, 36 V
o o o o
5)
+ 2+
6) Mg(s) + 2H (aq) → Mg (aq) + H2(g)

Question 6
1) a) Cl is reduced from +5 to +4, thus requires a reducing agent
b) S is reduced from +6 to -2, thus requiring a reducing agent.
c) Mn is oxidised from +2 to +4, thus an oxidising agent is required
d) Zn is oxidised from 0 to +2, thus an oxidising agent is required.
2) a) Cr is oxidised, and Sn is reduced
b) Hg is reduced, Fe is oxidised
c) As is oxidised, Cl is reduced.

Question 7
1) a) Reduction is defined as the gain of electrons (Remember the mnemonic: OIL RIG – oxidation is loss,
reduction is gain)
3+ 2+
b) The oxidising agent is itself reduced. In the reaction, Fe is reduced from Fe to Fe .
3+ - 2+
c) Fe + e → Fe
3+
d) H2O2 is a stronger reducing agent than Fe (the oxidising agent) in the Table of Standard Reduction
Potentials.
2) a) The table is a part of Table 4B of the Standard Reduction Potentials … the higher on the table an
element is, the stronger its reducing potential. Zn has the highest reduction potential of the substances
listed.

2+ -
b) (i) Fe → Fe + 2e
3+ -
(ii) Au + 3e → Au
3+ 2+
(iii) 3Fe + 2Au → 3Fe + 2Au
3) K2Cr2O7: potassium has an oxidation number of +1, oxygen of -2. The sum of all oxidation numbers must
be zero since the molecule is neutral .. thus
2(+1) + 2(Cr) + 7(-2) = 0, therefore the oxidation number of chromium is +6.
4) a) and b) Zinc (Zn) or chromium (Cr), or any of the metals with a stronger reducing ability than Fe (Mn, Al,
Mg, etc.) may be used for galvanising. This will ensure that the Fe is not oxidised.

Question 8
1) The hydrogen half-cell must operate at a pressure of 1 atm (101,3 kPa) and at a temperature of 25°C
(298 K).
2) Platinum is suitable since it is inert and a conductor of electricity.
3) a) Salt bridge
b) This is a galvanic cell, and the cathode is positive, the anode negative. To calculate the potential of the
X|X2+ half-cell: E   Ecathode  Eanode  0, 31  0  Eanode  Eanode  0, 31 V
o o o o

c) Reduction takes place at the cathode (at the positive electrode) .. in this case, the hydrogen half-cell.
+ -
The reaction: 2H + 2e → H2

pg 55
a) E   Ecathode  Eanode  2, 05  0, 31  Eanode  Eanode  2, 36 V
o o o o
4)
Thus M must be magnesium (Mg), with a reduction potential of -2,36 V.
b) The reaction is exothermic – energy is released. An endothermic reaction would require energy (in the
form of a battery) to proceed.
c) The cell has become flat – it has reached equilibrium, with no difference in potential between the two
half-cells.

Question 9
1) a) An electrolyte is a solution, liquid or dissolved substance that conducts electricity through the
movement of electrons.
+
b) H - the oxidation number of H decreases from +1 to 0.
- -
c) 2Cl → Cl2(g) + 2e
- -
HH2O + 2e → H2 + 2OH
- -
2Cl + 2H2O → Cl2 + H2 + 2OH or alternately: 2NaCl + 2H2O → 2NaOH + Cl2 + H2
2) a) Ni(s)
2+ -
b) Ni → Ni + 2e
c) Lower: E°cell = E°cathode – E°anode = 1,36 – (0,34) = 1,02 V.

Question 10
1) n(Cu) = ½ n(electrons)
23 23
number of atoms = n × NA = (0,8 × 0,5) × 6,022 × 10 = 2,408×10 atoms.
2) n = m / M, thus m(Cu) = n × M = 0,4 × 63,5 = 25,4 g
%Cu = 25,4 / 28 × 100 = 90,71%
The impure copper is not suitable since the percentage purity (90,71%) < 99,99%
2+
3) Cu is a stronger reducing agent tha Pt and Ag and will thus be oxidised to Cu .

pg 56
Grade 12
Exam Preparation

Chemical Systems
Memorandum

TouchTutor® Series
Physical Sciences

Section A (Multiple Choice Questions)


1) A – Nitrogen, phosphorus and potassium (N, P, K) are primary nutrients needed by plants.
2) D – nitrogen fertiliser; this is a source of macronutrients
3) B – micronutrients are necessary for plant health, but in smaller quantities than macronutrients – this is the
primary difference between macro- and micronutrients. The term ‘minor elements’ isn’t used very much
these days, the term micronutrients being preferred. Many of the micronutrients are indeed necessary for
the enzyme functions in plants, and for some micronutrients, there is a fine line between deficiency levels
and toxic levels.
4) B - P: 3/7 × 22/100 × 400 = 37,7 g
5) C - first calculate the mass of N, P and K in say, a 1 kg (1000 g) sample ..
N: 5/10 × 40/100 × 1000 = 200 g ― 200/14,0 = 14,29 mol of N
P: 2/10 × 40/100 × 1000 = 80 g ― 80/31,0 = 2,58 mol of P
K: 3/10 × 40/100 × 1000 = 120 g ― 120/39,1 = 3,07 mol of K
The mole ratio is 14,29 : 2,58 : 3,07, which, when simplified, is equal to 55 : 10: 12
6) D – phosphorus is needed to stimulate root growth
7) C – ammonia (NH3) is produced in the Haber process.
8) D – a high yield is achieved by promoting the forward reaction – a low temperature, and high pressure.
(note the difference between question 18 and 19)
9) D – commercially the Haber process takes place at 200 atm, i.e. a very high pressure

10) B – the oxidation state for nitrogen changes from –3 in NH3 (H is +1), to +2 in NO (O = -2), to +4 in N2O
(O = -2), to +5 in HNO3 (H = =1, O = -2)
11) A - this reaction is exothermic. Lowering the temperature will increase the value of K (the equilibrium
constant), and the yield of NO2. This reaction is unusual in that the yield increases as the temperature is
lowered. Enthalpy of reaction, ΔH, does not alter with temperature. It is dependent on the reactants and
products, and their states.
12) B - the yield (amount of product obtained) is not necessarily improved by a faster rate; in fact they are
often opposing factors due to equilibrium considerations. A catalyst will increase the rate of a reaction, but
not the yield.
13) C – vanadium pentoxide (V2O5), also called vanadium (V) oxide.
14) B – all the reactions are exothermic
15) A – W = heated air, X = ammonia gas, Y = air, Z = water – just follow the individual steps of the Ostwald
process. Note, W could also be heated oxygen. From a commercial perspective, it is cheaper to use air
rather than oxygen
16) C – allowing livestock to wade in and drink from a water source increases rather than diminishes runoff
into waterways.
17) C – adding nitrates would exacerbate the eutrophication, not remedy it.

Section B: Structured Questions – see next page

pg 57
Question 1
1) a) The chemical formula for oleum is H2S2O7 – the balanced equation: H2SO4 + SO3 → H2S2O7
b) The reaction is highly exothermic and produces a toxic mist.
2) a) Phosphate promote strong, healthy roots and so the farmer should choose ammonium phosphate
since it has the highest percentage of phosphate present
b) overuse of fertiliser may lead to change in soil quality and changes in natural habitats; with heavy rains
can lead to runoff from agricultural lands and contamination of water sources; in water bodies, can
lead to eutrophication and dead zones
Question 2
1) a) Gas A is nitrogen – derived from the fractional distillation of air …
b) Gas B is hydrogen – derived from coal gasification and natural gas reforming
Nitrogen and hydrogen react according to the equation N2(g) + 3H2(g) ⇌ 2NH3(g) to form ammonia.
c) The reaction of nitric acid with ammonia produces the fertiliser ammonium nitrate (NH4NO3)
2) a) Sulfuric acid is prepared in the contact process
b) 2NH3(g) + H2SO4(ℓ) ⇌ (NH4)2SO4 (s)
3) a) The Ostwald process refers to the whole process of forming nitric acid (HNO3) from ammonia. The
preparation of NO, nitrogen monoxide, from NH3 and O2 is the catalytic oxidation of ammonia. The
reaction takes place in the presence of platinum (Pt). It is a strongly exothermic reaction.
b) The balanced equation: 4NH3(g) + 5O2(g) → 4NO(g) + 6H2O(g).
c) when NO is reacted with oxygen, nitrogen dioxide (NO2) is formed.

Question 3
1) Primary nutrients: nitrogen (N), phosphorus (P), potassium (K)
2) a) Process P: fractional distillation
b) product X: hydrogen gas (H2)
c) product Y: ammonia gas (NH3(g))
d) NH3(ℓ) + HNO3(ℓ) → NH4NO3(s)
e) Ostwald process
3) a) Contact process
b) sulfuric acid (H2SO4)
4) rain may wash excess fertiliser into water bodies, contaminating them with excess nutrients leading to
eutrophication
Excessive growth of algae, alien vegetation, etc.
Excessive fertiliser seeps into groundwater, making the groundwater unsuitable for animals or plants,
causing the ‘blue-baby’ syndrome (excessive nitrate exposure)
Question 4
1) Organic fertiliser is derived from plant or animal matter, with naturally occurring minerals
2) When fertiliser is washed into water bodies, or is leached from the soil and enter rivers or streams, there is
an excess of nutrients in the water, promoting strong algae growth. The algae die and decompose,
depleting the oxygen in the water. This leads to the death of aquatic life dependent on the oxygen.
3) (NH4)2SO4

Question 5
1) Fractional distillation is the separation of a mixture into its component parts, i.e. its fractions.
2) Air is liquefied at -200°C. As it enters the fractioning column, the liquid nitrogen boils to form gas and rises
to the top and is extracted. Liquid oxygen collects at the bottom from where it is piped off.
3) Because both oxygen and nitrogen as in the gas phase at room temperature
4) a) Steam reforming: Steam reforming is a method for producing hydrogen, carbon monoxide, or other
useful products from hydrocarbon fuels such as natural gas. This is achieved in a processing device
called a reformer which reacts steam at high temperature with the fossil fuel.
b) a mixture of carbon monoxide (CO) and hydrogen (H2)
c) CH4(g) + H2O(g) → CO2(g) + 3H2(g)
Question 6
1) a) Process 1: fractional distillation of liquid air
b) N2(g) + 3H2(g) ⇌ 2NH3(g)
c) nitric acid (HNO3)
d) 2NH3(g) + H2SO4(ℓ) ⇌ (NH4)2SO4 (s)
e) nitrogen (N)
4) a) increases (note: the rate of production) – an increase in temperature will increase the rate of reaction
for both exo- and endothermic reactions.
b) the yield will decrease, since the reverse reaction is favoured.

pg 58
5)
Reaction SO2 O2 SO3
Mole ratio 2 1 2
Initial moles 4 x/32 0
Change (moles) -3 -1,5 3 (1,5 × 2)
Equilibrium
1 x/32 – 1,5 3
moles
Equil.
½ = 0,5 (x – 48)/64 3/2 = 1,5
concentration

SO  2
(1, 5)
2

Kc  3
  4, 5  x  176 g
SO  O  (0, 5)
2
2
2
2
( x  48) / 64

Alternately, make the initial moles of oxygen equal to x (rather than having its mass equal to x).
Then … (see next page)

Reaction SO2 O2 SO3


Mole ratio 2 1 2
Initial moles 4 x 0
Change (moles) -3 -1,5 3 (1,5 × 2)
Equilibrium
1 x – 1,5 3
moles
Equil.
½ = 0,5 (x – 1,5)/2 3/2 = 1,5
concentration

SO  2
(1, 5)
2

Kc  3
  4, 5  x  5, 5
SO   O  (0, 5) ( x  1, 5) / 2
2
2
2
2

Now m  nM  5, 5  32  176 g

Question 7
1) fractional distillation of air
2) N2(g) + 3H2(g) ⇌ 2NH3(g)
3) Ammonium sulfate (NH4)2SO4
4) V2O5 – vanadium pentoxide

The term ‘contact’ comes from the fact that the reaction takes place when passed over (when in contact
with) the catalyst.
5) one doesn’t know the exact composition – whether is has all the nutrients or micronutrients necessary
Manure has very variable composition, and generally has low levels of micronutrients
Food grown organically is more expensive
6) a) Only 26% of the contents are fertiliser, and of that, only 5 of 9 parts are potassium, i.e. 14,4 %
b) potassium is needed for the development of good quality flowers and fruit / it also keeps plants more
frost resistant.

pg 59

You might also like